Answer Key & Exp for GS Prelim Test-3 held on 2nd Dec’ 2018 · Commission report 1930, the 3...

74
Page 1 of 74 Answer Key & Exp for GS Prelim Test-3 held on 2 nd Dec’ 2018 1. Under Article 12 of the Indian Constitution, which of the following bodies come under the definition of ‘State’? (1) District Boards (2) Navratnas like ONGC (3) Any private agency working for AADHAR registration Select the correct answer using the code given below: (a) 1 and 2 only (b) 2 and 3 only (c) 1 and 3 only (d) 1, 2 and 3 Answer: (d) Explanation: Article 12 has defined the term ‘State’ for the purposes of Part III. According to it, the State includes the following: (a) The Government and the Parliament of India, that is, the Executive and the Legislative organs of the Union Government. Statements 1 and 2 are correct as per (b) and (c) given below: (b) The Government and Legislature of States, that is, the Executive and the Legislative organs of the State Government. (c) All local authorities i.e. Municipalities, Panchayats, District Boards, Improvement Trusts, etc. This provision is also given in Entry 5; List II of the 7 th Schedule to the Indian Constitution. (d) All other authorities, that is, statutory or non-statutory authorities like LIC, ONGC, SAIL, etc. The Board of Directors of a Central Public Sector Enterprises (CPSE) exercise delegated powers subject to broad policy guidelines issued by the Government. The categorization of PSUs has granted enhanced powers to the Boards of the profit making enterprises under various schemes like ‘Maharatna’, ‘Navratna’ and ‘Miniratna’ Statement 3 is correct: According to the Supreme Court interpretation in Ajay Hasia vs Khalid Mujhib Sehravardi case, 1980, even a private body or an agency working as an instrument of the State falls within the meaning of the ‘State’ under Article 12. Note: The Indian Constitution does not mention Judiciary under the definition of the ‘State’. Source: Vajiram & Ravi Institute Yellow Book Part-1 on Indian Polity; Ch.8: Fundamental Rights; Page No. 41 2. Which of the following provision of the Indian Constitution resembles the ‘Instrument of Instructions’ enumerated in the Government of India Act, 1935? (a) Fundamental Rights (b) Fundamental Duties (c) Directive Principles of State Policy

Transcript of Answer Key & Exp for GS Prelim Test-3 held on 2nd Dec’ 2018 · Commission report 1930, the 3...

Page 1: Answer Key & Exp for GS Prelim Test-3 held on 2nd Dec’ 2018 · Commission report 1930, the 3 Round Table Conferences and the White Paper on Constitutional Reforms. What is called

Page 1 of 74

Answer Key & Exp for GS Prelim Test-3 held on 2nd Dec’ 2018

1. Under Article 12 of the Indian Constitution, which of the following bodies come under the definition of ‘State’? (1) District Boards (2) Navratnas like ONGC (3) Any private agency working for AADHAR registration Select the correct answer using the code given below: (a) 1 and 2 only (b) 2 and 3 only (c) 1 and 3 only (d) 1, 2 and 3

Answer: (d) Explanation: Article 12 has defined the term ‘State’ for the purposes of Part III. According to it, the State includes the following: (a) The Government and the Parliament of India, that is, the Executive and the Legislative organs of the Union Government. Statements 1 and 2 are correct as per (b) and (c) given below: (b) The Government and Legislature of States, that is, the Executive and the Legislative organs of the State Government. (c) All local authorities i.e. Municipalities, Panchayats, District Boards, Improvement Trusts, etc. This provision is also given in Entry 5; List II of the 7th Schedule to the Indian Constitution. (d) All other authorities, that is, statutory or non-statutory authorities like LIC, ONGC, SAIL, etc. The Board of Directors of a Central Public Sector Enterprises (CPSE) exercise delegated powers subject to broad policy guidelines issued by the Government. The categorization of PSUs has granted enhanced powers to the Boards of the profit making enterprises under various schemes like ‘Maharatna’, ‘Navratna’ and ‘Miniratna’ Statement 3 is correct: According to the Supreme Court interpretation in Ajay Hasia vs Khalid Mujhib Sehravardi case, 1980, even a private body or an agency working as an instrument of the State falls within the meaning of the ‘State’ under Article 12. Note: The Indian Constitution does not mention Judiciary under the definition of the ‘State’. Source: Vajiram & Ravi Institute Yellow Book Part-1 on Indian Polity; Ch.8: Fundamental Rights; Page No. 41 2. Which of the following provision of the Indian Constitution resembles the

‘Instrument of Instructions’ enumerated in the Government of India Act, 1935? (a) Fundamental Rights (b) Fundamental Duties (c) Directive Principles of State Policy

Page 2: Answer Key & Exp for GS Prelim Test-3 held on 2nd Dec’ 2018 · Commission report 1930, the 3 Round Table Conferences and the White Paper on Constitutional Reforms. What is called

Page 2 of 74

(d) Preamble Answer: (c) Explanation: Directive Principles of State Policy is the correct answer. The Directive Principles of State Policy (DPSPs) are enumerated in Part IV of the Constitution from Articles 36 to 51. It has been borrowed from the Irish Constitution of 1937, which had been copied from the Spanish Constitution.

They resemble the ‘Instrument of Instructions’ enumerated in the Government of India Act, 1935. In the words of Dr B.R. Ambedkar, "the Directive Principles are like the instrument of instructions, which were issued to the Governor-General and to the Governors of the colonies of India by the British Government under the Government of India Act of 1935. The Government of India Act of 1935 was a result of the Nehru Report 1928, the Simon Commission report 1930, the 3 Round Table Conferences and the White Paper on Constitutional Reforms. What is called Directive Principles is merely another name for the Instrument of Instructions. The only difference is that they are instructions to the Legislature and the Executive." Hence, Option (c) is the correct answer.

In 1927, the INC resolved to set up a Committee to draft a "Swaraj Constitution" for India based on a declaration of rights that would provide safeguards against oppression. The 11-member Committee, led by Motilal Nehru, was constituted in 1928. Its report made a number of recommendations, including proposing guaranteed fundamental rights to all Indians. Several of these provisions were later replicated in various parts of the Indian Constitution including fundamental rights.

The Fundamental Duties are defined as the moral obligations of all citizens to help promote a spirit of patriotism and to uphold the unity of India. These duties, set out in Part IV–A of the Constitution concern individuals and the nation. The idea for Fundamental Duties has been taken from erstwhile USSR.

The Preamble to the Indian Constitution embodies the fundamental values and the philosophy on which the Constitution is based. Its sets out the main objectives which the Constituent Assembly intended to achieve.

Source: Indian Polity by M. Laxmikanth: 5th edition; Chapter-8: DPSP; Page No. 8.1 3. Consider the following statements:

(1) The residuary power of legislation is vested with the Parliament of India.

(2) In case of any conflict between the Union Law and the State Law (on a concurrent subject), the decision of the Supreme Court of India is final regarding which law among the two prevails.

(3) The division of subjects between the Union and the States under the Seventh Schedule can be altered by the Parliament through an amendment to the Constitution of India.

Page 3: Answer Key & Exp for GS Prelim Test-3 held on 2nd Dec’ 2018 · Commission report 1930, the 3 Round Table Conferences and the White Paper on Constitutional Reforms. What is called

Page 3 of 74

(4) There is a 4-fold division of subjects under the Seventh Schedule viz.: Union, State, Concurrent and Local Governments.

In the context of the distribution of legislative powers between Union and State governments; which of the statements given above are incorrect? (a) 1 and 2 only (b) 1 and 3 only (c) 2 and 3 only (d) 2 and 4 only

Answer: (d) Explanation: Statement 1 is correct: The residuary power of legislation is vested with the Parliament of India. Residuary subjects are the matters which are not enumerated in any of the three lists. Statement 2 is incorrect: In case of any conflict between the Union Law and the State Law (on a concurrent subject), the former prevails. However, the State Law will prevail if it has been reserved for the consideration of the President and has received her/his assent. Statement 3 is correct: The division of subjects between the Union and the States under the Seventh Schedule can be altered by the Parliament with an amendment to the Constitution. Certain subjects like Forests, Education have been moved from State List to Concurrent List via an amendment to the Constitution. Statement 4 is incorrect: There is only a 3-fold division of subjects under the Seventh Schedule viz.: Union, State and Concurrent. Local governments are not conferred legislative powers by the Indian Constitution. Source: Indian Polity by M. Laxmikanth: 5th edition, Ch. 14: Center-State Relations, Page 14.2. 4. With reference to the Parliament's power to reorganize the States under

Article 3 of the Indian Constitution’, which of the following statements is/are correct? (1) A bill contemplating the reorganisation of States can be introduced in

the Parliament with or without prior recommendation of the President. (2) The bill, after its introduction in the Parliament has to be referred to the

State Legislature concerned for expressing its views within a specified period.

Select the correct answer using the code given below: (a) 1 only (b) 2 only (c) Both 1 and 2 (d) Neither 1 nor 2

Answer: (d) Explanation: Article 3 of the Indian Constitution authorises the Parliament to:

Page 4: Answer Key & Exp for GS Prelim Test-3 held on 2nd Dec’ 2018 · Commission report 1930, the 3 Round Table Conferences and the White Paper on Constitutional Reforms. What is called

Page 4 of 74

(a) form a new State by separation of territory from any State or by uniting two or more States or parts of States or by uniting any territory to a part of any State, (b) Increase the area of any State, (c) Diminish the area of any State, (d) Alter the boundaries of any State, and (e) Alter the name of any State. However, Article 3 lays down two conditions in this regard: Statement 1 is incorrect: A bill contemplating the above changes can be introduced in the Parliament only with the prior recommendation of the President; Statement 2 is incorrect: Before recommending the bill, the President has to refer the same to the State Legislature concerned for expressing its views within a specified period.

The President (or the Parliament) is not bound by the views of the State Legislature and may either accept or reject them, even if the views are received in time.

Further, it is not necessary to make a fresh reference to the State Legislature every time an amendment to the bill is moved and accepted in the Parliament.

Source: Vajiram & Ravi Institute Yellow Book Part-1 on Indian Polity; Ch.6: Union and its Territory; Page No. 28. 5. By which of the following Act, the ‘Legislative Council of India’ received the

power to discuss the budget for the first time? (a) The Indian Councils Act, 1861 (b) The Indian Councils Act, 1892 (c) The Morley-Minto Reforms 1909 (d) The Government of India Act, 1935

Answer: (b) Explanation: The Indian Councils Act 1861 was passed by the British Parliament on 1st August 1861 to make substantial changes in the composition of the Governor General’s Council for executive & legislative purposes. The most significant feature of this Act was the association of Indians with the legislation work. Indian Councils Act 1892 was passed by the British Parliament to increase in the size of the Legislative Councils. This Act marked the beginning of the representative form of Government. The Indian Councils Act, 1892 authorized the increase in members of the Central and Provincial Legislative Councils but maintained the official majority in them. It gave them the power to discuss the budget and addressing questions to the Executive. Hence, option (b) is correct. The Indian Councils Act 1909 or Morley-Minto Reforms was passed by the British Parliament in 1909 in an attempt to widen the scope of Legislative Councils, placate the demands of moderates in Indian National Congress and to increase the participation of Indians in the administration.

Page 5: Answer Key & Exp for GS Prelim Test-3 held on 2nd Dec’ 2018 · Commission report 1930, the 3 Round Table Conferences and the White Paper on Constitutional Reforms. What is called

Page 5 of 74

The Government of India Act 1935 was passed by British Parliament in August 1935. This Act ended the system of dyarchy introduced by the Government of India Act 1919 and provided for the establishment of a Federation of India to be made up of Provinces of British India and some or all of the Princely States. However, the Federation never came into being as the required number of Princely States did not join it. Source: Indian Polity by M. Laxmikant-Ch.1: Historical Background; Page no 1.5, 1.6 6. With reference to representative democracy, consider the following

statements: (1) Social inequality undermines the proper functioning of representative

democracy. (2) Representative democracy is founded on the principle of

republicanism. Which of the statements given above is/are correct? (a) 1 only (b) 2 only (c) Both 1 and 2 (d) Neither 1 nor 2

Answer: (a) Explanation: In its standard meaning, representative democracy has four main features: (a) The sovereignty of the people expressed in the electoral appointment of

the representatives; (b) Representation as a free mandate relation; (c) Electoral mechanisms to ensure some measure of responsiveness to the

people by representatives who speak and act in their name; and (d) The universal franchise, which grounds representation as an important

element of political equality. Statement 1 is correct: While the last two are the tools for ensuring the political equality, representative democracy doesn’t have inbuilt mechanism to ensure fairness in its functioning wherein socio-economic equality is concerned. Social inequality is accompanied by disproportional distribution of power. This, in turn, affects the answerability and accountability of the representative towards the people. Statement 2 is incorrect: United Kingdom is a monarchy, having a representative government. Sources:

http://openjournals.maastrichtuniversity.nl/Marble/article/view/533

Vajiram & Ravi Institute Yellow Book; Indian Polity, Part 1, Page 8

https://www.aph.gov.au/About_Parliament/Senate/Powers_practice_n_procedures/pops/Papers_on_Parliament_66/The_Contemporary_Crisis_of_Representative_Democracy

Page 6: Answer Key & Exp for GS Prelim Test-3 held on 2nd Dec’ 2018 · Commission report 1930, the 3 Round Table Conferences and the White Paper on Constitutional Reforms. What is called

Page 6 of 74

7. Dr B.R. Ambedkar proudly acclaimed that the Constitution of India has been framed after ‘ransacking all the known Constitutions of the World’. In this context, consider the following pairs:

Provision/Feature of the Constitution

Source from where derived/ drawn

1. Emergency Provisions : Government of India Act, 1935

2. Method of election of the President : Australian Constitution

3. Joint sitting of the two Houses : Irish Constitution

4. Prerogative writs : British Constitution

Which of the pairs given above are correctly matched? (a) 1 and 3 only (b) 1 and 4 only (c) 2 and 3 only (d) 1, 3 and 4 only

Answer: (b) Explanation: The Constitution of India has borrowed most of its provisions from the constitutions of various other countries as well as from the Government of India Act of 1935. Sources of Features Borrowed: ● Government of India Act of 1935 - Federal Scheme, Office of the Governor,

Judiciary, Public Service Commissions, Emergency Provisions and Administrative details.

● British Constitution - Parliamentary Government, Rule of Law, Legislative procedure, Single citizenship, Cabinet system, Prerogative writs, Parliamentary privileges and Bicameralism.

● US Constitution - Fundamental Rights, Independence of Judiciary, Judicial Review, Impeachment of the President, Removal of the Supreme Court and High Court judges and post of the Vice-President.

● Irish Constitution - Directive Principles of State Policy, nomination of members to the Rajya Sabha and method of election of the President.

● Canadian Constitution - Federation with a strong Centre, vesting of residuary powers in the Centre, appointment of State Governors by the Centre, and advisory jurisdiction of the Supreme Court.

● Australian Constitution - Concurrent List, freedom of trade, commerce and inter-course and joint sitting of the two Houses of the Parliament.

● Weimar Constitution of Germany - Suspension of Fundamental Rights during Emergency.

Source: Indian Polity by M. Laxmikanth 5th edition; Ch.3: Salient Features of the Constitution; Table 3.4. 8. With reference to the ‘Parliamentary System in India’, consider the

following statements:

Page 7: Answer Key & Exp for GS Prelim Test-3 held on 2nd Dec’ 2018 · Commission report 1930, the 3 Round Table Conferences and the White Paper on Constitutional Reforms. What is called

Page 7 of 74

(1) The Constitution of India provides for the parliamentary form of government, both at the Centre and in the States.

(2) The Executive, for its policies and acts, is constitutionally independent of the Legislature.

(3) The Parliamentary form of government is also known as responsible government, which is prevalent in Maldives.

Which of the statements given above is/are correct? (a) 1 only (b) 1 and 2 only (c) 2 and 3 only (d) 1, 2 and 3

Answer: (a) Explanation: ● Statement 1 is correct: Articles 74 and 75 of the Indian Constitution deal

with the parliamentary system at the Centre and Articles 163 and 164 deal with the States.

● Statement 2 is not correct: The parliamentary government is also known as ‘responsible government’ as the cabinet (the real executive) is accountable (and not independent) to the Parliament (Lok Sabha specifically) and stays in office as long as it enjoys the latter’s confidence. Similarly, the State Cabinet is accountable to the Legislative Assembly of the State concerned.

● Statement 3 is not correct as the government of Maldives is Presidential Republic based on the Constitution adopted on August 7, 2008. The Parliamentary system on the other hand is prevalent in Britain, Japan, Canada, and India among others.

Sources:

Vajiram & Ravi Institute Yellow Book on Indian Polity; Part 1; Ch.5: Parliamentary and Presidential Systems of Government; Page No. 20, 21.

https://www.studycountry.com/guide/MV-government.htm

9. Which of the following fundamental rights are available to both citizens and foreigners alike in India? (1) Right to Primary Education (2) Freedom to manage religious affairs (3) Protection of language and culture Select the correct answer using the code given below: (a) 1 only (b) 1 and 2 only (c) 2 and 3 only (d) 1, 2 and 3

Answer: (b) Explanation:

Page 8: Answer Key & Exp for GS Prelim Test-3 held on 2nd Dec’ 2018 · Commission report 1930, the 3 Round Table Conferences and the White Paper on Constitutional Reforms. What is called

Page 8 of 74

Fundamental Rights available only to citizens are: ● Prohibition of discrimination on grounds of religion, race, caste, sex or place

of birth (Article 15). ● Equality of opportunity in matters of public employment (Article 16). ● Protection of six rights regarding freedom of : (i) speech and expression, (ii)

assembly, (iii) association, (iv) movement, (v) residence, and (vi) profession (Article 19)

● Protection of language, script and culture of minorities (Article 29). Hence statement 3 is incorrect.

● Right of minorities to establish and administer educational institutions (Article 30)

All others FR are available to both citizens and foreigners alike, except enemy aliens which are: ● Equality before law and equal protection of laws (Article 14). ● Protection in respect of conviction for offences (Article 20). ● Protection of life and personal liberty (Article 21). ● Right to elementary education (Article 21A). Hence statement 1 is correct. ● Protection against arrest and detention in certain cases (Article 22). ● Prohibition of traffic in human beings and forced labour (Article 23). ● Prohibition of employment of children in factories etc., (Article 24). ● Freedom of conscience and free profession, practice and propagation of

religion (Art. 25). ● Freedom to manage religious affairs (Article 26). Hence statement 2 is

correct. ● Freedom from payment of taxes for promotion of any religion (Article 27). ● Freedom from attending religious instruction or worship in certain educational

institutions (Article 28).

Source: Vajiram & Ravi Institute Yellow Book on Indian Polity; Part 1; Ch.8: Fundamental Rights; Page No. 45, 46. 10. With reference to the ‘emergency declared under Article 352 of the Indian

Constitution’, which of the following provisions are correct? (1) The resolution declaring emergency is required to be passed by a

special majority of the Lok Sabha under Article 368, within one month of such a proclamation.

(2) The emergency once proclaimed can be extended by 6 months at a time with no upper limit.

(3) The President can revoke the proclamation of emergency at any time.

Select the correct answer using the code given below: (a) 2 and 3 only (b) 1 and 2 only (c) 1 and 3 only (d) 1, 2 and 3

Page 9: Answer Key & Exp for GS Prelim Test-3 held on 2nd Dec’ 2018 · Commission report 1930, the 3 Round Table Conferences and the White Paper on Constitutional Reforms. What is called

Page 9 of 74

Answer: (a) Explanation: Under Article 352, the President can declare a national emergency when the security of India or a part of it is threatened by war or external aggression or armed rebellion. The President, can proclaim a national emergency only after receiving a written recommendation from the cabinet after the 44th Amendment Act.. Statement 1 is incorrect: The proclamation of Emergency must be approved by both the Houses of Parliament within one month from the date of its issue. Originally, the period allowed for approval by the Parliament was two months, but was reduced by the 44th Amendment Act of 1978. However, if the proclamation of emergency is issued at a time when the Lok Sabha has been dissolved or the dissolution of the Lok Sabha takes place during the period of one month without approving the proclamation, then the proclamation survives until 30 days from the first sitting of the Lok Sabha after its reconstitution, provided the Rajya Sabha has in the meantime approved it. Statement 2 is correct: If approved by both the Houses of Parliament, the emergency continues for six months, and can be extended to an indefinite period with an approval of the Parliament for every six months. This provision for periodical parliamentary approval was also added by the 44th Amendment Act of 1978. Statement 3 is correct: A proclamation of emergency may be revoked by the President at any time by a subsequent proclamation. Such a proclamation does not require the parliamentary approval. Further, the President must revoke a proclamation if the Lok Sabha passes a resolution disapproving its continuation. Again, this safeguard was introduced by the 44th Amendment Act of 1978. Before the amendment, a proclamation could be revoked by the President himself and the Lok Sabha had no control in this regard. Source: Vajiram & Ravi Institute yellow book, Part - 1, Ch. 13: Emergency Provisions; Page No.111 11. Which of the following Directive Principles of State Policy is/are considered

as Gandhian Principles? (1) To organise agriculture and animal husbandry on modern lines. (2) To promote the educational and economic interests of SCs, STs and

other weaker sections of the society. (3) To make provision for just and humane conditions of work and

maternity relief. (4) To organise village panchayats. Select the correct answer using the code given below: (a) 4 only (b) 1 and 4 only (c) 2 and 4 only (d) 1, 2 and 4 only

Page 10: Answer Key & Exp for GS Prelim Test-3 held on 2nd Dec’ 2018 · Commission report 1930, the 3 Round Table Conferences and the White Paper on Constitutional Reforms. What is called

Page 10 of 74

Answer: (c) Explanation: The Directive Principles of State Policy (DPSPs) are enumerated in Part IV of the Constitution from Articles 36 to 51. The Constitution of India does not contain any classification of Directive Principles. However, on the basis of their content and direction, they can be classified into three broad categories, viz, socialistic, Gandhian and liberal–intellectual. Gandhian Principles: These principles are based on Gandhian ideology. They represent the programme of reconstruction enunciated by Gandhi during the national movement. They require the State to:

Article 40: To organise village panchayats and endow them with necessary powers and authority to enable them to function as units of self-government. Hence, statement 4 is correct.

Article 46: To promote the educational and economic interests of SCs, STs, and other weaker sections of the society and to protect them from social injustice and exploitation. Hence, statement 2 is correct.

Statement 1 is incorrect: “Organizing agriculture and animal husbandry on modern and scientific lines (Article 48)” is considered as Liberal-Intellectual Principle. Statement 3 is incorrect: “To make provision for just and humane conditions of work and maternity relief (Article 42)” is considered as Socialistic Principle. Source: Indian Polity- M. Laxmikanth: 5th edition; Ch-8: DPSP; Page No.8.2 12. The Parliament of India can legislate on a subject enumerated in the ‘State

List of Seventh Schedule to the Indian Constitution’ under which of the following conditions? (1) When a resolution is passed by the Rajya Sabha with a special

majority. (2) To implement an International Agreement. (3) When the State is under the President’s Rule. (4) When Legislatures of two or more States request the Parliament to

make a law on a matter enumerated in the State List. Select the correct answer using the code given below: (a) 1 and 2 only (b) 1, 2 and 4 only (c) 2, 3 and 4 only (d) 1, 2, 3 and 4

Answer: (d) Explanation: The Parliament of India can legislate on a subject enumerated in the ‘State List of Seventh Schedule to the Indian Constitution’ under the following conditions:

Page 11: Answer Key & Exp for GS Prelim Test-3 held on 2nd Dec’ 2018 · Commission report 1930, the 3 Round Table Conferences and the White Paper on Constitutional Reforms. What is called

Page 11 of 74

Statement 1 is correct: Under Article 249 if the Council of States passes a resolution supported by not less than two-thirds of the members present and voting that it is necessary in the national interest that the Parliament should make laws with respect to any matter enumerated in the State List, then the Parliament can make laws with respect to that matter while the resolution remains in force. Statement 2 is correct: Under Article 252 if two or more States pass resolution requesting the Parliament to enact laws on matters listed in the State List, then the Parliament can make laws on those subjects. Statement 3 is correct: Under Article 253 the Parliament can make laws on a State subject to implement international treaties. It enables the Union government to fulfil its obligations under international agreements. Statement 4 is correct: Under Article 356 when the President’s rule is imposed in a State, the Parliament becomes empowered to make laws with respect to any matter in the State List with respect to that State. Source: Indian Polity by M. Laxmikanth: 5th edition, Ch. 14: Centre-State Relations, Page No. 14.3-14.4. 13. Consider the following statements:

(1) The term ‘Union of India’ includes only States while the term ‘Territory of India’ includes not only the States but also the Union Territories.

(2) The Indian Constitution provides for the admission or establishment of new States that are not part of the Union of India by amending the Constitution under Article 368.

Which of the statements given above is/are correct? (a) 1 only (b) 2 only (c) Both 1 and 2 (d) Neither 1 nor 2

Answer: (a) Explanation: Statement 1 is correct: The ‘Territory of India’ is a wider expression than the ‘Union of India’ because the latter includes only States while the former includes not only the States but also Union Territories and territories that may be acquired by the Government of India at any future time. The States are the members of the federal system and share distribution of powers with the Centre. The Union Territories and the acquired territories, on the other hand, are directly administered by the Central government. Being a sovereign state, India can acquire foreign territories according to the modes recognised by international law, i.e., cession (following treaty, purchase, gift, lease or plebiscite), occupation (hitherto unoccupied by a recognised ruler), conquest or subjugation. Statement 2 is incorrect: Article 2 empowers the Parliament to ‘admit into the Union of India, or establish, new States on such terms and conditions as it thinks

Page 12: Answer Key & Exp for GS Prelim Test-3 held on 2nd Dec’ 2018 · Commission report 1930, the 3 Round Table Conferences and the White Paper on Constitutional Reforms. What is called

Page 12 of 74

fit’. Thus, Article 2 grants two powers to the Parliament: (a) the power to admit into the Union of India new States; and (b) The power to establish new States. Article 2 relates to the admission or establishment of new States that are not part of the Union of India. Article 3, on the other hand, relates to the formation of or changes in the existing States of the Union of India. In other words, Article 3 deals with the internal re-adjustment inter se of the territories of the constituent States of the Union of India. Source: Indian Polity by M. Laxmikanth; Ch.5: Union and its Territory; Page No. 5.2 14. Consider the following statements:

(1) The Council of Ministers is collectively responsible to the Parliament of India.

(2) A person can be a Minister without being a Member of the Parliament.

(3) The oath of secrecy to the ministers is administered by the Prime Minister of India.

Which of the statements given above is/are incorrect? (a) 1 only (b) 2 only (c) 1 and 3 only (d) 1, 2 and 3

Answer: (c) Explanation: ● Statement 1 is not correct: The Indian Constitution under Article 75(3)

states that the Council of Ministers shall be collectively responsible to the House of the People (Lok Sabha) and not the Parliament.

● Statement 2 is correct: The Constitution stipulates that a Minister who is not a Member of the Parliament for a period of six consecutive months, ceases to be a Minister.

● Statement 3 is not correct: The ministers operate on the principle of secrecy of procedure and cannot divulge information about their proceedings, policies and decisions. They take the oath of secrecy before entering their office. However, the oath of secrecy to the ministers is administered by the President of India and not the Prime Minister.

Vajiram & Ravi Institute Yellow Book-Part 1 on Indian Polity; Ch.5: Parliamentary and Presidential Systems of Government; Page No. 20, 21. 15. The 97th Constitutional Amendment Act of 2011 gave constitutional status

and protection to Cooperative Societies. In this context, consider the following statements: (1) It brought about changes in Fundamental Rights, Directive Principles

Page 13: Answer Key & Exp for GS Prelim Test-3 held on 2nd Dec’ 2018 · Commission report 1930, the 3 Round Table Conferences and the White Paper on Constitutional Reforms. What is called

Page 13 of 74

of State Policy as well as added a part to the Constitution of India. (2) The term of office-bearers and elected members has been fixed by

the Constitution itself. (3) The superintendence, direction and control of the preparation of

electoral rolls and the conduct of elections to a co-operative society shall vest in the State Election Commission.

Which of the statements given above is/are incorrect? (a) 3 only (b) 1 and 2 only (c) 1 and 3 only (d) 1, 2 and 3

Answer: (a) Explanation: The 97th Constitutional Amendment Act 2001 made the following changes in the Constitution: ● It made the right to form co-operative societies a fundamental right (Article

19(1)). ● It included a new Directive Principle of State Policy on promotion of co-

operative societies (Article 43-B). ● It added a new Part IX-B in the Constitution which is entitled “The Co-

operative Societies” (Articles 243-ZH to 243-ZT). Hence, statement 1 is correct. Other important provisions of the Act are: ● The term of office of elected members of the board and its office bearers shall

be five years from the date of election. Hence, statement 2 is correct. ● The superintendence, direction and control of the preparation of electoral rolls

and the conduct of elections to a co-operative society shall vest in such body, as may be provided by the State Legislature. Hence, statement 3 is incorrect.

Source: Indian Polity by M. Laxmikanth 5th edition; Ch.3 and 60: Salient Features of the Constitution & Co-operative Societies; Page No. 3.12 and 60.1 16. Consider the following provisions in the Indian Constitution:

(1) Formation of new States and alteration of areas, boundaries or names of the existing States

(2) Privileges of the Parliament, its members and its Committees (3) Conferment of more jurisdictions on the Supreme Court (4) Elections to the Parliament and the State Legislatures (5) The Fifth Schedule and Sixth Schedule - administration of Scheduled

Areas Which of the provisions given above can be amended by simple majority of the Parliament? (a) 1 and 2 only

Page 14: Answer Key & Exp for GS Prelim Test-3 held on 2nd Dec’ 2018 · Commission report 1930, the 3 Round Table Conferences and the White Paper on Constitutional Reforms. What is called

Page 14 of 74

(b) 1, 2 and 5 only (c) 1, 2 and 3 only (d) 1, 2, 3, 4 and 5

Answer: (d) Explanation: Article 368 provides for two types of amendments firstly, by a special majority of Parliament and secondly, special majority along with ratification by half of the State Legislatures by a simple majority. But, some other articles provide for the amendment of certain provisions of the Constitution by a simple majority of Parliament, that is, a majority of the members of each House present and voting (similar to the ordinary legislative process). Notably, these amendments are not deemed to be amendments of the Constitution for the purposes of Article 368. These provisions include:

1. Admission or establishment of new States. 2. Formation of new States and alteration of areas, boundaries or

names of existing States. 3. Privileges of the Parliament, its members and its Committees. 4. Conferment of more jurisdictions on the Supreme Court. 5. Elections to the Parliament and the State Legislatures. 6. Fifth Schedule—administration of Scheduled Areas and Scheduled

Tribes. 7. Sixth Schedule—administration of tribal areas.

As, all the provisions provided can be amended by simple majority. Hence, all the given provisions are correct. Sources:

Vajiram & Ravi Institute Yellow Book, Indian Polity, Part 1, Ch.11; Page 103

Indian Polity, M Laxmikanth, 5th Edition, Ch.10: Amendment of the Indian Constitution; Page 10.3

17. Regarding ‘Effect of National Emergency on Fundamental Rights’,

consider the following provisions in the Constitution of India: (1) In case of proclamation of National Emergency due to war or external

aggression, the fundamental rights under Article 19 are automatically suspended.

(2) Enforcement of all other fundamental rights in Part III of the Constitution can be suspended by a Presidential order to that effect.

Which of the statements given above is/are correct? (a) 1 only (b) 2 only (c) Both 1 and 2 (d) Neither 1 nor 2

Page 15: Answer Key & Exp for GS Prelim Test-3 held on 2nd Dec’ 2018 · Commission report 1930, the 3 Round Table Conferences and the White Paper on Constitutional Reforms. What is called

Page 15 of 74

Answer: (a) Explanation: Statement 1 is correct: According to Article 358, when a proclamation of national emergency is made, the six Fundamental Rights under Article 19 are automatically suspended. No separate order for their suspension is required. The 44th Amendment Act of 1978 restricted the scope of Article 358 in two ways. Firstly, the six Fundamental Rights under Article 19 can be suspended only when the National Emergency is declared on the ground of war or external aggression and not on the ground of armed rebellion. Secondly, only those laws which are related with the Emergency are protected from being challenged and not other laws. Also, the executive action taken only under such a law is protected. Statement 2 is incorrect: Article 359 authorizes the president to suspend the right to move any court for the enforcement of Fundamental Rights during a National Emergency i.e. right to constitutional remedy is suspended. This means that under Article 359, the Fundamental Rights as such are not suspended, but only their enforcement. The said rights are theoretically alive but the right to seek remedy is suspended. The suspension of enforcement relates to only those Fundamental Rights that are specified in the Presidential Order and not all other Fundamental Rights. E.g. Article 359 does not empower the suspension of enforcement of the Fundamental Rights under Articles 20 and 21. Source: Vajiram & Ravi Institute yellow book, Part - I, Ch. 13: Emergency Provisions; Page No.112 18. The Indian Constituent Assembly was established under:

(a) The Government of India Act, 1935 (b) The Cripps Mission, 1942 (c) The Cabinet Mission, 1946 (d) The Indian Independence Act, 1947

Answer: (c) Explanation: The Government of India Act,1935 : This Act ended the system of dyarchy introduced by the Government of India Act 1919 and provided for establishment of a Federation of India to be made up of Provinces of British India and some or all of the Princely States., The Cripps Mission, 1942: Cripps Mission was sent by the British Government in March 1942 to India with key objective to secure Indian cooperation and support for British war efforts. Headed by Sir Stafford Cripps, this mission sought to negotiate an agreement with Indian leaders. The Indian Constituent Assembly was proposed in 1934 by M.N. Roy and it became official demand of the Indian National Congress in 1935. Under the Cabinet Mission Plan of 1946, elections were held for the first time for the Constituent Assembly. The Constitution of India was drafted by the Constituent

Page 16: Answer Key & Exp for GS Prelim Test-3 held on 2nd Dec’ 2018 · Commission report 1930, the 3 Round Table Conferences and the White Paper on Constitutional Reforms. What is called

Page 16 of 74

Assembly, and it was implemented under the Cabinet Mission Plan on 16 May 1946. Hence, option (c) is correct. Indian Independence Act, 1947: It was based upon the Mountbatten plan of 3rd June 1947 and was passed by the British Parliament on July 5, 1947. It received royal assent on July 18, 1947. Source: Indian Polity by M. Laxmikant-Ch.1: Historical Background; Page no. 2.1 19. The Constitution of India declares that all ‘laws’ inconsistent with or in

derogation of the Fundamental Rights shall be void. The term 'law' here doesn't include which of the following? (a) Constitutional Amendment Acts (b) Ordinances issued by the President (c) Rules and regulations not passed by the Legislature (d) Customs having the force of law

Answer: (a) Explanation: The term ‘law’ in Article 13 has been given a wide connotation so as to include the following: (a) Permanent laws enacted by the Parliament or the state legislatures; (b) Temporary laws like ordinances issued by the President or the state

Governors; Hence option (b) is incorrect. (c) Statutory instruments in the nature of delegated legislation (executive

legislation) like order, bye-law, rule, regulation or notification. Hence option (c) is incorrect.

(d) Non-legislative sources of law, that is, custom or usage having the force of law. Hence option (d) is incorrect.

Thus, not only legislation but any of the above can be challenged in the courts as violating a Fundamental Right and hence, can be declared null and void. Further, Article 13 declares that a constitutional amendment is not a 'law' in that sense and hence can amend the Constitution. The Supreme Court held in the Kesavananda Bharati case (1973) that a Constitutional amendment can be challenged on the ground that it violates a fundamental right that forms a part of the ‘basic structure’ of the Constitution. But it still doesn't come under the definition of 'law.' Hence option (a) is correct. Source: Vajiram & Ravi Institute Yellow Book on Indian Polity; Part 1; Ch.8: Fundamental Rights; Page No. 43. 20. Consider the following pairs:

Directive Principles Origin

1. Protection and improvement of : The 44th Constitutional Amendment Act, 1978

Page 17: Answer Key & Exp for GS Prelim Test-3 held on 2nd Dec’ 2018 · Commission report 1930, the 3 Round Table Conferences and the White Paper on Constitutional Reforms. What is called

Page 17 of 74

environment

2. Equal Justice and free legal aid : The 42nd Constitutional Amendment Act, 1976

3. Promotion of Cooperative Societies : The 86th Constitutional Amendment Act, 2002

Which of the pairs given above is/are correctly matched? (a) 2 only (b) 1 and 2 only (c) 1 and 3 only (d) 1, 2 and 3

Answer: (a) Explanation: Pair 1 is incorrectly matched: Protection and improvement of environment and to safeguard forests and wild life (Article 48 A) was added through 42nd Constitutional Amendment Act, 1976 and not the 44th Constitutional Amendment Act, 1978. Pair 2 is correctly matched: To promote equal justice and free legal aid to the poor (Article 39A) was added through 42nd Constitutional Amendment Act, 1976. Pair 3 is incorrectly matched: Promotion of Cooperative societies (Article 43B) was added through 97th Constitutional Amendment Act, 2011.

The 44th Constitutional Amendment Act of 1978 added one more Directive Principle, which requires the State to minimise inequalities in income, status, facilities and opportunities (Article 38).

The 86th Constitutional Amendment Act of 2002 changed the subject-matter of Article 45 and made elementary education a fundamental right under Article 21 A. The amended directive requires the State to provide early childhood care and education for all children until they complete the age of six years.

The 97th Constitutional Amendment Act of 2011 added a new Directive Principle relating to co-operative societies. It requires the state to promote voluntary formation, autonomous functioning, democratic control and professional management of co-operative societies (Article 43B).

Source: Vajiram & Ravi Institute Yellow Book on Indian Polity; Part 1; Ch.9: Directive Principles of State Policy; Page No. 93, 94. 21. Which of the following are ‘Extra-Constitutional’ devices for promoting

Centre-State coordination? (1) National Institution for Transforming India (2) Inter-State Council (3) Zonal Councils (4) Finance Commission of India Select the correct answer using the code given below: (a) 1 and 2 only (b) 1 and 3 only

Page 18: Answer Key & Exp for GS Prelim Test-3 held on 2nd Dec’ 2018 · Commission report 1930, the 3 Round Table Conferences and the White Paper on Constitutional Reforms. What is called

Page 18 of 74

(c) 2 and 3 only (d) 3 and 4 only

Answer: (b) Explanation:

Inter-State Council (Article 263) and Finance Commission (Article 280) are Constitutional bodies as provisions related to their formation and functioning are provided in the Constitution itself. The Inter State Council is formed by the President with a purpose to discuss or investigate policies, subjects of common interest, and disputes among states. It is headed by the Prime Minister of India.

Union Finance Commission is constituted by the President of India under Article 280 every five years. It is formed to define the financial relations between the central government of India and the individual state governments.

NITI Aayog (National Institution for Transforming India) and Zonal Councils are extra Constitutional bodies. NITI Aayog is an extra-constitutional non-statutory body created by Executive. It is a policy think tank of the Government of India, established with the aim to achieve Sustainable Development Goals and to enhance cooperative federalism by fostering the involvement of State Governments of India in the economic policy-making process using a bottom-up approach.

Zonal Councils are statutory bodies created under State Reorganization Act 1956 and the North Eastern Council Act, 1971. These are advisory councils and are made up of the states of India that have been grouped into six zones to foster cooperation among them.

Source: Indian Polity by M. Laxmikanth: 5th edition, Ch. 14: Centre-State Relations, Page 14.8 22. Consider the following pairs:

List-I List-II

1. Persons of Indian Origin (PIO)

: An Indian citizen who is ordinarily residing outside India and holds an Indian Passport

2. Overseas Citizen of India (OCI)

: A foreigner registered so under the Citizenship Act, 1955

3. Non Resident Indian (NRI)

: A person who or any of whose ancestors was an Indian national and who is presently holding another country’s citizenship / nationality i.e. he/she is holding a foreign passport.

Which of the pairs given above is/are correctly matched? (a) 1 and 3 only (b) 2 only (c) 3 only

Page 19: Answer Key & Exp for GS Prelim Test-3 held on 2nd Dec’ 2018 · Commission report 1930, the 3 Round Table Conferences and the White Paper on Constitutional Reforms. What is called

Page 19 of 74

(d) None of the above Answer: (b) Explanation:

NRI: non-resident Indian An Indian citizen who is ordinarily residing outside India and holds an Indian Passport. An Indian citizen residing outside India for a combined total of at least 183 days in a financial year (from April 1 to March 31).

PIO : person of Indian origin Person who or whose any of ancestors was an Indian national and who is presently holding another country’s citizenship / nationality i.e. he/she is holding foreign passport

OCI : Overseas citizen of India A person registered so under the Citizenship Act, 1955. The following groups of people cannot have OCI status:

i. Anyone who was ever a citizen of Pakistan or Bangladesh ii. Anyone whose parents or grandparents were citizens of Afghanistan,

Pakistan, Bangladesh, China, or Sri Lanka iii. Anyone who served in a foreign military or worked in a foreign defence

department Source: Vajiram & Ravi Institute Yellow Book on Indian Polity; Part 1; Ch.7: Citizenship; Page No. 35, 36. 23. Which of the following political scientist called Indian Federalism as a

‘Cooperative Federalism’? (a) Ivor Jennings (b) Morris Jones (c) Granville Austin (d) Alexandrowicz

Answer: (c) Explanation: Option (c) is correct: Granville Austin stated that the Constituent Assembly produced a new kind of federalism to meet India’s peculiar needs. But ‘India’s peculiar needs’ have compounded manifolds thus posing a formidable challenge of enlisting the desired cooperation from all three – the Centre, the States and the local – sets of government. Hence, for efficient and effective working of the three layers of the Government cooperative federalism is essential in India. Ivor Jennings has described Indian Federalism as a “federation with a strong centralising tendency”. Morris Jones termed Indian federalism as a “bargaining federalism”. Alexandrowicz stated that “India is a case sui generis ( i.e., unique in character). Source:

Page 20: Answer Key & Exp for GS Prelim Test-3 held on 2nd Dec’ 2018 · Commission report 1930, the 3 Round Table Conferences and the White Paper on Constitutional Reforms. What is called

Page 20 of 74

Indian Polity by M. Laxmikant ; 5th edition; Ch.13: Federal System; Page 13.5

http://yojana.gov.in/Final%20Yojana%20Febuary%202015%20GS.pdf 24. The Indian Constitution envisages certain independent bodies as the

bulwarks of the democratic system of Government in India. Which among the following is not one of them? (a) The Election Commission of India (b) National Human Rights Commission (c) Comptroller and Auditor General of India (d) State Public Service Commissions

Answer: (b) Explanation: The Indian Constitution not only provides for the legislative, executive and judicial organs of the government (Central and state) but also establishes certain independent bodies. They are envisaged by the Constitution as the bulwarks of the democratic system of Government in India. These are: (a) Election Commission to ensure free and fair elections to the Parliament, the state legislatures, the office of President of India and the office of Vice-president of India. (b) Comptroller and Auditor-General of India to audit the accounts of the Central and state governments. He acts as the guardian of public purse and comments on the legality and propriety of government expenditure. (c) Union Public Service Commission to conduct examinations for recruitment to all-India services and higher Central services and to advise the President on disciplinary matters. (d) State Public Service Commission in every state to conduct examinations for recruitment to state services and to advice the governor on disciplinary matters. Hence, option (b) is the correct answer. The Constitution ensures the independence of these bodies through various provisions like security of tenure, fixed service conditions, expenses being charged on the Consolidated Fund of India, and so on. Source: Indian Polity by M. Laxmikanth 5th edition; Ch.3: Salient Features of the Constitution; Page No. 3.5 25. Consider the following pairs:

Constitutional Amendment Provisions Added

1. The First Amendment 1951 : Ninth Schedule

2. The Seventy-Third Amendment 1992 : Panchayati Raj System

3. The Fifty-second Amendment 2014 : Anti Defection Law

Which of the pairs given above is/are correctly matched?

Page 21: Answer Key & Exp for GS Prelim Test-3 held on 2nd Dec’ 2018 · Commission report 1930, the 3 Round Table Conferences and the White Paper on Constitutional Reforms. What is called

Page 21 of 74

(a) 2 only (b) 1 and 2 only (c) 2 and 3 only (d) 1, 2 and 3

Answer: (d) Explanation: Pair 1 is correctly matched: The First Amendment of the Constitution of India, enacted in 1951, provided against abuse of freedom of speech and expression, validation of zamindari abolition laws, and clarified that the right to equality does not bar the enactment of laws which provide "special consideration" for weaker sections of society. This Amendment set the precedent of amending the Constitution to overcome judicial judgements impeding fulfilment of the government's perceived responsibilities to particular policies and programmes. It inserted Ninth Schedule to the Constitution to protect the land reform and other laws present in it from the judicial review. It also inserted Articles 31A and 31B. Pair 2 is correctly matched: The 73rd Amendment 1992 added a new Part IX to the constitution titled “The Panchayats” covering provisions from Article 243 to 243(O); and a new Eleventh Schedule covering 29 subjects within the functions of the Panchayats. This amendment implements the Article 40 of the DPSP which says that “State shall take steps to organise village panchayats and endow them with such powers and authority as may be necessary to enable them to function as units of self-government”. Pair 3 is correctly matched: 52nd Constitutional Amendment Act 1985 Anti Defection Law - Provide disqualification of members from parliament and assembly in case of defection from one party to other. However, parts of the 10th Schedule to the Constitution of India was struck down by the Supreme Court in the case of Kihoto Hollohan v. Zachillhu 1992 ,for being in contravention with Article 368 of the Constitution. Source: Indian Polity, M Laxmikanth, 5th Edition, Appendix VI: Constitutional Amendments at a Glance; Page no: AVI.2-AVI.9 26. Match List I with List II and select the correct answer using the code given

below the lists.

List-I List-II

A. Establishment of the Supreme Court

1. Regulating Act, 1773

B. Establishment of Board of Control 2. Government of India Act, 1919

C. Introduction of Bicameralism 3. Government of India Act, 1935

D. Introduction of Federal System 4. Pitts India Act, 1784

Codes A B C D (a) 2 3 1 4 (b) 1 4 2 3

Page 22: Answer Key & Exp for GS Prelim Test-3 held on 2nd Dec’ 2018 · Commission report 1930, the 3 Round Table Conferences and the White Paper on Constitutional Reforms. What is called

Page 22 of 74

(c) 1 2 3 4 (d) 4 3 2 1

Answer: (b) Explanation: Regulating Act of 1773 provided for the establishment of a Supreme Court at Calcutta comprising of one Chief Justice and three other judges. Pitts India Act of 1784 empowered the Board of control to supervise and direct all operations of the civil and military government or revenues of the British possessions in India. Government of India Act, 1919 introduced Bicameralism .The Indian Legislative Council was replaced by a bicameral legislature consisting of an Upper House and a Lower House. The Government of India Act 1935 provided for the establishment of an All India Federation made up of both princely and provincial states. It divided the powers between the Centre and provincial legislatures and abolished dyarchy in the provinces, giving them autonomy and introduced dyarchy at the Centre. Source: Indian Polity by Laxmikant-Ch.1: Historical Background; Page 1.1. – 1.6 – 1.7 27. With reference to the ‘President’s Rule under Article 356’ of the Indian

Constitution, which of the following provisions are correct? (1) The 42nd Constitutional Amendment Act 1976 amended Article 356

and provided that approval of the Parliament will be needed for one year at a time during President’s Rule.

(2) The 44th Constitutional Amendment Act 1978 introduced certain conditions for proclamation of President’s Rule beyond one year but maximum upto three years.

(3) The power vested in the President under President’s Rule is open to judicial review.

Select the correct answer using the code given below: (a) 1 and 2 only (b) 1 and 3 only (c) 2 and 3 only (d) 1, 2 and 3

Answer: (d) Explanation: All the given provisions are correct. The 44th Amendment Act of 1978 introduced a new provision to put restraint on the power of Parliament to extend a proclamation of President’s Rule beyond one year. Thus, it provided that, beyond one year, the President’s Rule can be extended by six months at a time only when the following two conditions are fulfilled: 1. A proclamation of National Emergency should be in operation in the whole

of India, or in the whole or any part of the state.

Page 23: Answer Key & Exp for GS Prelim Test-3 held on 2nd Dec’ 2018 · Commission report 1930, the 3 Round Table Conferences and the White Paper on Constitutional Reforms. What is called

Page 23 of 74

2. The Election Commission must certify that the general elections to the legislative assembly of the concerned state cannot be held on account of difficulties.

The 38th Amendment Act of 1975 made the satisfaction of the President in invoking Article 356 final and conclusive which could not be challenged in any court on any ground. But, this provision was subsequently deleted by the 44th Amendment Act of 1978 implying that the satisfaction of the President is not beyond judicial review. Source: Vajiram & Ravi Institute yellow book, Part 1, Ch. 13: Emergency Provisions; Page No.114, 115. 28. Consider the following terms/phrases:

(1) Martial Law (2) Judicial Review (3) Essential Religious Practice Which of the above terms is/are not mentioned in the Constitution? (a) 3 only (b) 2 and 3 only (c) 1 and 2 only (d) 1, 2 and 3

Answer: (b) Explanation:

Article 34: Notwithstanding anything in the foregoing provisions of this Part, Parliament may by law indemnify any person in the service of the Union or of a State or any other person in respect of any act done by him in connection with the maintenance or restoration of order in any area within the territory of India where martial law was in force or validate any sentence passed, punishment inflicted, forfeiture ordered or other act done under martial law in such area. Hence, Martial Law finds mention in the Constitution.

Judicial review is a process under which executive or legislative actions are subject to review by the judiciary. Article 13 declares that all the laws that are inconsistent with or in derogation of any of the fundamental rights shall be void. Thus, Judicial Review is implicit under Article 13 as it has conferred ‘doctrine of judicial review’ expressively to the Supreme Court under Article 32 and the High Court under Article 226 of the Constitution. Thus, it is not mentioned explicitly anywhere in the Constitution.

A seven-judge Bench of the Supreme Court invented the doctrine of “essentiality” in the Shirur Mutt case in 1954. The court held that the term “religion” will cover all rituals and practices “integral” to a religion, and took upon itself the responsibility of determining the essential and non-essential practices of a religion. The essentiality/integrality doctrine has tended to lead the court into an area that is beyond its competence (Judicial Overreach), and given judges the power to decide purely religious questions. Essential Religious Practice is a doctrine founded by the Supreme Court for testing

Page 24: Answer Key & Exp for GS Prelim Test-3 held on 2nd Dec’ 2018 · Commission report 1930, the 3 Round Table Conferences and the White Paper on Constitutional Reforms. What is called

Page 24 of 74

whether some religious practice finds security under Article 25 of the Constitution. It is not mentioned anywhere in the Constitution.

Sources:

Indian Polity by M. Laxmikanth 5th Edition, Ch. 7 :Fundamental Rights, Page No.7.21

https://indianexpress.com/article/explained/ayodhya-case-supreme-court-verdict-mosque-integral-to-islam-ismail-faruqui-judgement-5377466/

29. Which of the following pairs is/are correctly matched?

Body Chairperson/ Head

1. Inter-State Council : The Prime Minister of India

2. Zonal Councils : Union Home Minister

3. Finance Commission of India

: The President of India

Select the correct answer using the code given below: (a) 1 only (b) 1 and 2 only (c) 2 and 3 only (d) 1, 2 and 3

Answer: (b) Explanation:

Inter State Council can be established by the President under Article 263. The Prime Minister of India is the chairperson of the Inter State Council.

Zonal Councils are extra-Constitutional statutory bodies created through Parliamentary Acts to foster inter-state cooperation. Union Home Minister is the Chairperson of all the Zonal Councils.

The Finance Commission of India is constituted by the President under Article 280. Its Chairperson is appointed by the Government of India. Article 280 mandates that the Chairperson must be a person having ‘experience in public affairs’. Hence, pairs 1 and 2 are correct; pair 3 is incorrect.

Source: Indian Polity by M. Laxmikanth: 5th edition, Ch. 14: Centre-State Relations, Page 14.11 30. With reference to establishment, composition and duties of the ‘Inter-State

Council’ in India, consider the following statements: (1) It can be established by the Prime Minister of India in public interest. (2) The President of India is empowered to define the duties of an Inter-

State Council (3) It consists of the Prime Minister and Chief Ministers of all the States

Page 25: Answer Key & Exp for GS Prelim Test-3 held on 2nd Dec’ 2018 · Commission report 1930, the 3 Round Table Conferences and the White Paper on Constitutional Reforms. What is called

Page 25 of 74

only. Which of the statements given above is/are correct? (a) 2 only (b) 1 and 3 only (c) 2 and 3 only (d) 1, 2 and 3

Answer: (a) Explanation: Statement 1 is incorrect: Under Article 263, the President of India can establish such a council if at any time it appears to him that the public interest would be served by its establishment. Statement 2 is correct: The President of India can define the nature of duties to be performed by such a council and its organisation and procedure. Statement 3 is incorrect: The composition of the Council is as follows: (i) The Prime Minister as the Chairman (ii) The Chief Ministers of all the States (iii) The Chief Ministers of Union Territories having Legislative Assemblies (iv) The Administrators of Union Territories not having Legislative Assemblies (v) The Governors of States under President’s rule (vi) Six Central cabinet ministers, including the Home Minister, to be nominated

by the Prime Minister. Five Ministers of Cabinet rank / Minister of State (independent charge) nominated by the Chairman of the Council (i.e., Prime Minister) are permanent invitees to the Council. Source: Vajiram & Ravi Institute Yellow Books Polity-II; Ch. 4.4; Page No. 75 31. Consider the following statements:

(1) The Fundamental Rights under Articles 14, 19 and 31 of the Constitution are completely subordinated to all the Directive Principles.

(2) The provisions given in the ‘Directive Principles of State Policy’ are the only directives enumerated in the Indian Constitution which are non-justiciable.

Which of the statements given above is/are correct? (a) 1 only (b) 2 only (c) Both 1 and 2 (d) Neither 1 nor 2

Answer: (d) Explanation: Statement 1 is incorrect: The Parliament by the 25th Constitutional Amendment Act 1971, introduced a new Article 31-C which stated that if the

Page 26: Answer Key & Exp for GS Prelim Test-3 held on 2nd Dec’ 2018 · Commission report 1930, the 3 Round Table Conferences and the White Paper on Constitutional Reforms. What is called

Page 26 of 74

State enacts any law giving effect to the Directive principles namely, Article 39(a) and Article 39(b) and in the process if the law violates the Fundamental Rights enumerated in Article 14, 19 and 31 of the Constitution, such laws cannot be held void for impinging the three Fundamental Rights and questioned in any court of law. It was challenged before the Supreme Court in the Keshavananda Bharati case in 1973. The court upheld the 25th Amendment Act, but struck down the 2nd part on the ground that the judicial review is a part of the ‘basic structure’ of the Constitution Parliament again by 42nd Constitutional Amendment Act, 1976, amended Article 31C to extend it to include “all” the Directive Principles. Thus, Article 14, 19 and 31 were completely subordinated to Directive Principles. In Minerva Mills case 1980, the change made under 42nd Amendment Act in Article 31-C was declared by the Supreme Court as unconstitutional on the ground that it affected the balance between Part III and Part IV (Principle of harmonization) of the Constitution. The court thus restored Article 31-C to its original state as interpreted in Keshavananda Bharati case. The present position is that ‘only’ Article 39(b) and Article 39(c) can be given precedence over Articles 14 and 19 and not all the Directive Principles (Article 31 Right to property was scrapped and made a constitutional right under Article 300A). Statement 2 is incorrect: Apart from the Directives included in Part IV, there are some other Directives contained in other Parts of the Constitution. They are:

Claims of SCs and STs to Services (Article 335 in Part XVI)

Instruction in mother tongue (Article 350-A in Part XVII)

Development of the Hindi Language (Article 351 in Part XVII) The above Directives are also non-justiciable in nature. However, they are also given equal importance and attention by the judiciary on the ground that all parts of the constitution must be read together. Source: Vajiram and Ravi Institute Yellow Book: Indian Polity Part-I: Chapter 9; Page No. 98 32. Which of the following have emerged as ‘basic features’ of the Indian

Constitution as per various judgments of the Supreme Court of India? (1) Welfare State (Socio-economic justice) (2) Secular character of the Constitution (3) Effective access to justice (4) Independence of the constitutional bodies Select the correct answer using the code given below: (a) 2 only (b) 1 and 2 only (c) 1, 2 and 3 only (d) 1, 2, 3 and 4

Answer: (c) Explanation:

Page 27: Answer Key & Exp for GS Prelim Test-3 held on 2nd Dec’ 2018 · Commission report 1930, the 3 Round Table Conferences and the White Paper on Constitutional Reforms. What is called

Page 27 of 74

The Parliament of India under Article 368 can amend any part of the Constitution including the Fundamental Rights but without affecting the ‘basic structure’ of the Constitution. However, the Supreme Court is yet to define or clarify as to what constitutes the ‘basic structure’ of the Constitution. From the various judgments, the following have emerged as ‘basic features’ of the Constitution or elements / components / ingredients of the ‘basic structure’ of the constitution: ● Supremacy of the Constitution ● Sovereign, democratic and republican nature of the Indian polity ● Secular character of the Constitution ● Separation of powers between the legislature, the executive and the judiciary ● Federal character of the Constitution ● Unity and integrity of the nation ● Welfare state (socio-economic justice) ● Judicial review ● Freedom and dignity of the individual ● Parliamentary system ● Rule of law ● Harmony and balance between Fundamental Rights and Directive Principles ● Principle of equality ● Free and fair elections ● Independence of Judiciary ● Limited power of Parliament to amend the Constitution ● Effective access to justice ● Principle of reasonableness ● Powers of the Supreme Court under Articles 32, 136, 141 and 142 Hence, option (c) is the correct alternative. Source: Vajiram and Ravi Institute Yellow Book: Indian Polity Part-1 Pg. 109 33. With reference to the ‘Charter Act of 1853’, which of the following

statements are correct? (1) It provided for the establishment of Indian (Central) Legislative

Council. (2) It introduced an open competition system of selection and

recruitment of civil servants. (3) The Macaulay Committee was appointed in 1854 as per the

provisions of the above Act. Select the correct answer using the code given below: (a) 1 and 2 only (b) 1 and 3 only (c) 2 and 3 only (d) 1, 2 and 3

Answer: (d) Explanation:

Page 28: Answer Key & Exp for GS Prelim Test-3 held on 2nd Dec’ 2018 · Commission report 1930, the 3 Round Table Conferences and the White Paper on Constitutional Reforms. What is called

Page 28 of 74

Statements (1), (2) and (3) are correct: Charter Act of 1853 established a separate Governor-General’s legislative council called Indian (Central) Legislative Council. This legislative wing of the council functioned as a mini-Parliament. Charter Act of 1853 introduced an open competition system of selection and recruitment of civil servants. The covenanted civil service was thus open to Indians also. Accordingly, Macaulay Committee (the committee on Indian Civil service) was appointed in 1854. Source: Indian Polity by Laxmikant-Ch.1: Historical Background; Page no 1.3 34. Members of the ‘Constituent Assembly’ who drafted the Constitution of

India were: (a) Nominated by the British Parliament (b) Elected by the Legislative Assemblies of various Provinces (c) Elected by the people (d) Nominated by the Governor General of India

Answer: (b) Explanation: The representatives of each community were to be elected by members of that community in the provincial legislative assembly and voting was to be done by the method of proportional representation by means of single transferable vote. The representatives of princely states were to be nominated by the heads of the princely states. The members were to be indirectly elected by the members of the provincial assemblies who themselves were elected on a limited franchise. The people of various Provinces elected the Legislative Assemblies by the direct election and members of these elected Legislative Assemblies elect the members of Constituent Assembly. Hence, option (b) is correct. Source: Indian Polity by M. Laxmikant; Ch.2: Making of the Constitution-Page no. 2.3 35. In the context of ‘Governor’s Rule in Jammu & Kashmir (J&K)’, which of the

following statements is/are correct? (1) In J&K, Governor’s Rule can be imposed for failure of the

constitutional machinery in the State. (2) The Governor, by proclamation, may subsume to himself all or any of

the functions of the Government of the State and all or any of the powers vested in or exercisable by anybody or any authority in the State.

Select the correct answer using the code given below: (a) 1 only (b) 2 only (c) Both 1 and 2 (d) Neither 1 nor 2

Page 29: Answer Key & Exp for GS Prelim Test-3 held on 2nd Dec’ 2018 · Commission report 1930, the 3 Round Table Conferences and the White Paper on Constitutional Reforms. What is called

Page 29 of 74

Answer: (c) Explanation: Since J&K has a separate Constitution, the Governor’s rule is imposed under Section 92 for six months after an approval by the President. In case the Assembly is not dissolved within six months, President’s rule under Article 356 is extended to the State. At present the State is under Governor's rule with the Assembly having been dissolved in November 2018. Statement 1 is correct: The imposition of Governor’s rule in J&K is slightly different than that in other States. In other States, the President’s rule is imposed under the Article 356 of Constitution of India. In J&K, governor’s rule is mentioned under Article 370 section 92 – ‘Provisions in case of failure of constitutional machinery in the State.’ Statement 2 is correct: Article 370 section 92 mentions provisions in case of failure of constitutional machinery in the State: If at any time, the Governor is satisfied that a situation has arisen in which the Government of the State cannot be carried on in accordance with the provisions of this Constitution, the Governor may by Proclamation: Assume to himself all or any of the functions of the Government of the

State and all or any of the powers vested in or exercisable by anybody or authority in the State.

Make such incidental and consequential provisions as appear to the Governor to be necessary or desirable for giving effect to the objects of the Proclamation, including provisions for suspending in whole or in part the operation of any provision of this Constitution relating to anybody or authority in the State.

.Sources:

Source: Vajiram & Ravi Institute Yellow Book on Indian Polity; Part 2; Ch.4.2: Provisions for Jammu & Kashmir; Page No. 61, 62.

https://www.thehindu.com/news/national/other-states/jk-all-set-for-presidents-rule/ article25532678.ece

36. Which of the following rulings of the Supreme Court of India or High Courts

was/were meant to uphold the Fundamental Rights? (1) Allowing women entry into Sabarimala Temple (2) Decriminalization of Adultery. Select the correct answer using the code given below: (a) 1 only (b) 2 only (c) Both 1 and 2 (d) Neither 1 nor 2

Answer: (c) Explanation: Statement 1 is correct. The Supreme Court of India has struck down a rule that disallowed girls and women in the 10-50 years age group from entering the

Page 30: Answer Key & Exp for GS Prelim Test-3 held on 2nd Dec’ 2018 · Commission report 1930, the 3 Round Table Conferences and the White Paper on Constitutional Reforms. What is called

Page 30 of 74

Sabarimala temple in Kerala. Chief Justice Dipak Misra-headed Constitution bench in a 4-1 verdict said the temple rule violated their right to equality (under Article 14) and right to worship (under Article 25) of the Constitution. Statement 2 is correct. The Supreme Court of India’s Constitution Bench argued, though ‘Decriminalization of Adultery’ affects the matrimonial sanctity aspect, but the provisions enacted counter to Article 14 (Right to Equality of the Constitution). Sources:

https://timesofindia.indiatimes.com/india/supreme-court-says-penal-provision-on-adultery-violative-of-right-to-equality/articleshow/65241875.cms

https://economictimes.indiatimes.com/news/politics-and-nation/supreme-court-allows-women-to-enter-sabarimala-temple/articleshow/65989807.cms

37. Which of the following Office holders are given a fixed two-year tenure in

their postings? (1) Cabinet Secretary (2) Foreign Secretary (3) Director, Research and Analysis Wing (4) Defence Secretary Select the correct answer using the code given below: (a) 1, 2 and 3 only (b) 1, 2 and 4 only (c) 2, 3 and 4 only (d) 1, 2, 3 and 4

Answer: (d) Explanation: Apart from the above, the Director, Intelligence Bureau; the Union Home Secretary; the Director, CBI; the DGP of States also enjoy a fixed two-year tenure. However, among the Central government-level officers, the Director, CBI's fixed tenure's sanctity is mandated by the Supreme Court and Others' fixed tenure was decided by the Central government. Source: Class Lectures 38. Which of the following laws exist for the implementation of the

‘Fundamental Duties’? (1) The Protection of Civil Rights Act, 1955 (2) The Prevention of Insults to National Honour Act, 1971 (3) The Representation of People Act, 1951 (4) The Wildlife (Protection) Act, 1972 Select the correct answer using the code given below: (a) 2 and 4 only (b) 1, 2 and 4 only (c) 2, 3 and 4 only

Page 31: Answer Key & Exp for GS Prelim Test-3 held on 2nd Dec’ 2018 · Commission report 1930, the 3 Round Table Conferences and the White Paper on Constitutional Reforms. What is called

Page 31 of 74

(d) 1, 2, 3 and 4 Answer: (d) Explanation: The Verma Committee on Fundamental Duties of the Citizens (1999) identified the existence of legal provisions for the implementation of some of the Fundamental Duties. They are mentioned below: 1. The Protection of Civil Rights Act (1955) provides for punishments for

offences related to caste and religion. 2. The Prevention of Insults to National Honour Act (1971) prevents

disrespect to the Constitution of India, the National Flag and the National Anthem.

3. The Representation of People Act (1951) provides for the disqualification of members of the Parliament or a state legislature for indulging in corrupt practice, that is, soliciting votes on the ground of religion or promoting enmity between different sections of people on grounds of caste, race, language, religion and so on.

4. The Wildlife (Protection) Act of 1972 prohibits trade in rare and endangered species.

Hence, (d) is the correct answer. Source: Indian Polity- M. Laxmikanth: Chapter-9: Fundamental Duties; Page No. 9.4 39. Consider the following statements:

(1) The Zonal Councils are constitutionally mandated to achieve an emotional integration of the country.

(2) The Inter-State Council has exclusive jurisdiction in deciding issues relating to inter-state, centre–state and centre–union territories relations.

Which of the statements given above is/are incorrect? (a) 1 only (b) 2 only (c) Both 1 and 2 (d) Neither 1 nor 2

Answer: (c) Explanation: Statement 1 is incorrect: The Zonal Councils are statutory and not constitutional bodies. Hence, their mandate of emotional integration is legal and not constitutional one. They are established by an Act of the Parliament, that is, States Reorganisation Act of 1956. The Act divided the country into five zones (Northern, Central, Eastern, Western and Southern) and provided a Zonal Council for each zone. While forming these zones, several factors have been taken into account which includes: the natural divisions of the country, the river

Page 32: Answer Key & Exp for GS Prelim Test-3 held on 2nd Dec’ 2018 · Commission report 1930, the 3 Round Table Conferences and the White Paper on Constitutional Reforms. What is called

Page 32 of 74

systems and means of communication, the cultural and linguistic affinity and the requirements of economic development, security and law and order. Statement 2 is incorrect: The Inter-State Council’s function to enquire and advise upon inter-state disputes is complementary to the Supreme Court’s jurisdiction under Article 131 to decide a legal controversy between the governments. The Council can deal with any controversy whether legal or non-legal, but its function is advisory unlike that of the Supreme Court which gives a binding decision. Source: Vajiram & Ravi Institute Yellow Book on Indian Polity; Part 2; Ch.4.4: Centre-State Relations ; Page No. 75, 76. 40. Secular character of the Indian State is evident from which of the following

provisions of the Constitution? (1) Preamble of the Indian Constitution (2) Fundamental Duties (3) Fundamental Rights Select the correct answer using the code given below: (a) 1 and 2 only (b) 2 and 3 only (c) 1 and 3 only (d) 1, 2 and 3

Answer: (d) Explanation: The Indian Constitution embodies the positive concept of secularism i.e. all religions in our country (irrespective of their strength) have the same status and equal support from the State. The Western concept of secularism connotes a complete separation between the religion (the church) and the state (the politics). The term ‘secular’ was added to the Preamble of the Indian Constitution by the 42nd Constitutional Amendment Act of 1976. Secular state finds its connotation in Fundamental rights (Articles 25-30: Right to freedom of religion; Cultural and educational rights). According to Fundamental duties enshrined under Article 51 A (e) of the Constitution, it shall be duty of every citizen of India to: promote harmony and spirit of common brotherhood amongst all the people of India transcending religious, linguistic and regional or sectional diversities. Hence, the correct answer code is (d). Source: Vajiram & Ravi Institute Yellow Book on Indian Polity; Part 2; Ch.3: Salient Features of the Constitution; Page No. 11 to 14.

Page 33: Answer Key & Exp for GS Prelim Test-3 held on 2nd Dec’ 2018 · Commission report 1930, the 3 Round Table Conferences and the White Paper on Constitutional Reforms. What is called

Page 33 of 74

41. With reference to ‘Continental Shelf’, consider the following statements: (1) It is the shallowest part of the ocean. (2) It provides rich sources of fisheries, petroleum and natural gas. (3) It has deposited sediments known as oozes. Which of the statements given above is/are correct? (a) 1 only (b) 1 and 2 only (c) 2 and 3 only (d) 1, 2 and 3

Answer: (b) Explanation: Continental shelf is one of the three sub-divisions of Continental Margin (the other two are Continental Slope and Continental Rise).

Statement 1 is correct. Continental shelf is the submerged part of the land adjoining the coast, shallow seas and gulfs. It is the shallowest part of the ocean showing an average gradient of 1 ֯ or even less. The width of the continental shelf varies from 60 km to as wide as 1300 kms. Statement 2 is correct. Continental shelf’s shallowness enables sunlight to penetrate through the water, which encourages the growth of microscopic organisms such as planktons, on which millions of fish and polyps survive. The Continental shelves are therefore, the richest fishing grounds in the world e.g., Grand Bank, off Newfoundland in North America; and Dogger Bank in the North Sea. Apart from minerals, sand and gravels, continental shelves are great sources of petroleum and natural gas e.g., Bombay High, Persian Gulf, North Sea, Gulf of Florida, etc. Statement 3 is incorrect. The sediments formed from the remains of living things are called oozes. They are pelagic sediments deposited in the deep sea/abyssal plains. They are divided into two groups on the basis of lime and silica contents - Calcareous oozes and Siliceous oozes. Source: Yellow book - Concept of Geography & Natural Resources, Chapter - 17, Page no. 158 42. In the context of oceanography, the term ‘Guyot’ refers to:

(a) A variety of sea-mount (b) A kind of coral reef

Page 34: Answer Key & Exp for GS Prelim Test-3 held on 2nd Dec’ 2018 · Commission report 1930, the 3 Round Table Conferences and the White Paper on Constitutional Reforms. What is called

Page 34 of 74

(c) A type of marine sediment (d) None of the above

Answer: (a) Explanation: A guyot is a flat-topped variety of a sea-mount occurring mainly in the Pacific Ocean. Their summits are almost entirely at depths of more than 1000 m from ocean water surface but rise up to 3 km from the ocean floor. The conical shape of the guyot has suggested to some scientists a volcanic origin, with the table-like summit having been planed down by marine erosion and finally submerged by a rise of sea level. Their summits are covered by sediments ranging from cretaceous sediments to volcanic materials of Tertiary age. Many guyots at the margin of oceans and continent are much deformed, tilted and broken at location of trenches/ocean margin. Their existence is one of the evidences of movement of plates (sea-floor spreading). The guyots must have been formed on the slope of the ridge but as the sea-floor spreads out, the guyots get located at the deep ocean floor.

Source: Yellow book - Concept of Geography & Natural Resources, Chapter – 17, Page no. 160 43. Which of the following factors affect ‘Oceanic Salinity’?

(1) Influx of river water (2) Precipitation (3) Rate of evaporation (4) The degree of water mixing by ocean currents. (5) Atmospheric pressure and wind direction Select the correct answer using the code given below: (a) 1, 2 and 3 only (b) 1, 3 and 4 only (c) 1, 3, 4 and 5 only (d) 1, 2, 3, 4 and 5

Answer: (d) Explanation: Ocean salinity is the term used to define the total content of the dissolved salts in ocean water. It is calculated as the amount of salt (in gm) dissolved in 1000

Page 35: Answer Key & Exp for GS Prelim Test-3 held on 2nd Dec’ 2018 · Commission report 1930, the 3 Round Table Conferences and the White Paper on Constitutional Reforms. What is called

Page 35 of 74

gm of sea water. It is usually expressed as parts per thousand or ppt. The average salinity of oceans is 35 ppt but it varies spatially and temporally in different oceans, seas and lakes. Factors affecting oceanic salinity:

Evaporation: There is direct positive relationship between the rate of evaporation and salinity e.g. greater the evaporation, higher the salinity and vice-versa. Salinity is higher near the tropics than at the equator. Both these regions, record high rate of evaporation but there is presence of dry air over the tropics of Cancer and Capricorn which helps in an efficient evaporation and subsequent increase in salinity, especially in the top layers of ocean water.

Ice formation: Formation of ice in the high latitude regions of the ocean increases seawater salinity.

Precipitation: It is inversely related to salinity e.g. higher the precipitation, lower the salinity and vice-versa. This is why the regions of high rainfall (equatorial zone) record comparatively lower salinity than the regions of low rainfall (sub-tropical high pressure belts).

Influx of river water: Though the rivers bring salt from the land to the oceans, big and voluminous rivers pour down immense volume of water into the oceans and thus salinity is reduced at their mouths e.g. comparatively low salinity is found near the mouths of the Ganga, the Congo, the Niger, etc. There is seasonal variation of surface salinity with maximum and minimum run-off from the land.

Atmospheric pressure and wind direction: Anti-cyclonic conditions with stable air and high temperature increase salinity of the surface water of the oceans. Sub-tropical high pressure belts represent such conditions to cause high salinity. Winds also help in the redistribution of salt in the oceans and the seas because winds drive away more saline water to less saline areas resulting into decrease of salinity in the former and increase in the latter.

Circulation of ocean water: Ocean currents affect the spatial distribution of salinity by mixing sea waters. Equatorial warm current drive away salts from western coastal areas of the continents and accumulate them along the eastern coastal areas.

Source: Yellow book - Concept of Geography & Natural Resources, Chapter – 17, Page no. 163 44. With reference to ‘Storm Surge’, consider the following statements:

(1) Storm surges can be caused due to gravitational pull of the Sun and the Moon.

(2) Coastal wetlands act as natural barrier in reducing the damage done by a storm surge.

Which of the statements given above is/are correct? (a) 1 only (b) 2 only (c) Both 1 and 2

Page 36: Answer Key & Exp for GS Prelim Test-3 held on 2nd Dec’ 2018 · Commission report 1930, the 3 Round Table Conferences and the White Paper on Constitutional Reforms. What is called

Page 36 of 74

(d) Neither 1 nor 2 Answer: (b) Explanation: A storm surge is a rise in sea level that occurs during tropical cyclones or intense storms. The storms produce strong winds that push the water into shore, which can lead to flooding. This makes storm surges very dangerous for coastal regions. Statement 1 is incorrect. Tides occur due to gravitational pull of the Sun and the Moon whereas a storm surge is primarily caused by the relationship between the winds and the ocean’s surface. The water level rises where the winds are strongest. Water is pushed in the direction in which the winds are blowing. The rotation of the Earth causes air to deflect toward the right in the Northern Hemisphere and toward the left in the Southern Hemisphere - a phenomenon known as the Coriolis Effect. If a cyclone develops in the Northern Hemisphere, the surge will be largest in the right-forward part of the storm. In the Southern Hemisphere, the surge will be largest in the left-forward part of the cyclone. Another factor contributing to storm surge is atmospheric pressure. The pressure is higher at the edges of a cyclone than it is at the centre. This pushes down the water in the outer parts of the storm, causing the water to bulge at the eye and eye wall—where the wind adds to the rise in sea level. Statement 2 is correct. Wetlands, such as swamps, estuaries, and mud flats, act as sponges for tropical cyclones. As the cyclone makes landfall, the marshy land and plants absorb the water and the energy of the storm surge. Silt and swamp vegetation prevent the most intense part of the storm surge from hitting homes and businesses. Coastal residents can reduce the damage done by a storm surge by protecting local wetlands. Source: Yellow book - Concept of Geography & Natural Resources, Chapter – 17, Page no. 165 45. Consider the following ocean currents of ‘North Pacific Gyre’:

(1) Kuro Shio current (2) North Pacific current (3) North Equatorial current (4) California current Select the correct sequence (clockwise) of the ocean currents using the code given below: (a) 1-2-3-4 (b) 3-1-2-4 (c) 3-1-4-2 (d) 2-3-4-1

Answer: (b) Explanation:

Page 37: Answer Key & Exp for GS Prelim Test-3 held on 2nd Dec’ 2018 · Commission report 1930, the 3 Round Table Conferences and the White Paper on Constitutional Reforms. What is called

Page 37 of 74

The North Pacific Gyre (NPG) is in the northern Pacific Ocean. It is located between the equator and 50° N latitude. The gyre has a clockwise circular pattern and is formed by four prevailing ocean currents:

North Pacific Current to the north;

California Current to the east;

North Equatorial Current to the south; and

Kuroshio Current to the west. Value addition: This site has unusual intense collection of man-induced marine debris known as the Great Pacific Garbage patch.

Source: Yellow book - Concept of Geography & Natural Resources, Chapter – 17, Page no. 167, 169 46. Consider the following factors:

(1) Submarine earthquake (2) Underwater landslide (3) Volcanic eruption (4) Impact of meteorite on ocean floor Which of the above factors can trigger the tsunamis in the ocean? (a) 1 and 2 only (b) 1 and 3 only (c) 1, 2 and 3 only (d) 1, 2, 3 and 4

Answer: (d) Explanation:

Page 38: Answer Key & Exp for GS Prelim Test-3 held on 2nd Dec’ 2018 · Commission report 1930, the 3 Round Table Conferences and the White Paper on Constitutional Reforms. What is called

Page 38 of 74

Tsunami, (Japanese: “harbour wave”), also called as seismic sea wave, is a catastrophic ocean wave, usually caused by a submarine earthquake, an underwater or coastal landslide, or a volcanic eruption. It can also occur due to the impact of meteorite(s). The term tidal wave is frequently used for such a wave, but it is a misnomer, for the wave has no connection with the tides. When an earthquake occurs; a train of simple, progressive oscillatory waves travels a great distance over the ocean surface in ever-widening circles. In deep water, a tsunami can travel as fast as 800 km (500 miles) per hour. The wavelengths are enormous, about 100 to 200 km (60 to 120 miles), but the wave amplitudes (heights) are very small, only about 30 to 60 cm (1 to 2 feet). The waves’ periods (the lengths of time for successive crests or troughs to pass a single point) are very long, varying from five minutes to more than an hour. These long periods, coupled with the extremely low steepness and height of the waves, enables them to be completely obscured in deep water by normal wind waves and swell. A ship in the high seas experiences the passage of a tsunami as an insignificant rise and fall of only half a metre (1.6 feet), lasting from five minutes to an hour or more. As the waves approach shoreline and enter shallower water they slow down and begin to grow in energy and height. The tops of the waves move faster than their bottoms do, which causes them to rise precipitously. Coastal waters may rise as high as 30 metres (about 100 feet) above normal sea level in 10 to 15 minutes. The continental shelf waters begin to oscillate after the rise in sea level. Between three and five major oscillations generate most of the damage. Oscillations may continue for several days until the ocean surface reaches equilibrium. Source: Yellow book - Concept of Geography & Natural Resources, Chapter – 17, Page no. 171, 172 https://www.britannica.com/science/tsunami 47. Match List-I with List-II and select the correct answer using the code given

below the lists:

List - I (Volcanic type)

List - II (Feature)

A. Shield volcanoes 1. Lava pit

B. Cinder Cones 2. Violent explosion

C. Lava dome 3. Highly viscous lava

Page 39: Answer Key & Exp for GS Prelim Test-3 held on 2nd Dec’ 2018 · Commission report 1930, the 3 Round Table Conferences and the White Paper on Constitutional Reforms. What is called

Page 39 of 74

D. Caldera 4. Gently rising slope with flat domes

Codes A B C D (a) 2 1 4 3 (b) 2 4 3 1 (c) 4 2 3 1 (d) 1 2 3 4

Answer: (c) Explanation: A volcano is an opening in the Earth’s surface where molten rock can escape from underneath. The Earth’s surface is made up of tectonic plates, which are spreading apart, crunching into each other, or sliding beside one another. Volcanoes are typically found at the fault lines between these plates. Some important volcanic landforms: Shield volcanoes are large, broad volcanoes that look like shields from above – hence the name. The lava that pours out of shield volcanoes is thin, so it can travel for great distances down the shallow slopes of the volcano. Cinder cones are the simplest type of volcano. They occur when particles and blobs of lava are ejected from a volcanic vent. The lava is blown violently into the air, and the pieces rain down around the vent. Lava domes are created by small masses of lava which are too viscous (thick) to flow very far. Unlike shield volcanoes, with low-viscosity lava, the magma from volcanic domes just piles up over and around the vent. Caldera is the lava pit into which the active vent pours hot lava. Composite volcanoes or strato-volcanoes make up some of the world’s most memorable mountains: Mount Rainier, Mount Fuji, and Mount Cotopaxi, for example. These volcanoes have a conduit system inside them that channels magma from deep within the Earth to the surface. Value addition: https://www.universetoday.com/27333/types-of-volcanoes/ Source: G C Leong - Certificate Human and Physical Geography, Chapter 3 - Page 22 48. The groundwater stored in the rock is released onto the surface at points

where the water table reaches the surface. In this context, what do you mean by a ‘Vauclusian Spring’? (a) In areas of tilted strata, where permeable and impermeable rocks

alternate, water emerges at the base of the permeable layers. (b) In Karst regions, water flows through passages in the rock by solution

re-emerging when limestone gives place to some impermeable rock. (c) When a dyke or sill of impermeable rocks is intruded through

permeable rocks.

Page 40: Answer Key & Exp for GS Prelim Test-3 held on 2nd Dec’ 2018 · Commission report 1930, the 3 Round Table Conferences and the White Paper on Constitutional Reforms. What is called

Page 40 of 74

(d) Water emerging at the foot of the scarp in chalk escarpments. Answer: (b) Explanation: There are several types of springs: In areas of tilted strata, where permeable and impermeable rocks alternate; water emerges at the base of the permeable layers. In well jointed rocks, water comes through the joints. In limestone or Chalk escarpments, water issues at the foot of the scarp as Scrap-foot spring. The correct option is (b). In Karst regions, river often disappears underground. Such river water flows through passages in the rock by solution re-emerging when limestone gives place to some impermeable rock. They are called as Vauclusian Springs or Resurgence. Source: G C Leong - Certificate Human and Physical Geography, Chapter 4 - Page 35 49. Which of the following create favourable conditions for the formation of

‘Deltas’? (1) Sea adjoining the delta should be shallow. (2) The coast should be favourable to tides. (3) Strong currents running at right angles to the river mouths Select the correct answer using the code given below: (a) 1 only (b) 1 and 3 only (c) 2 and 3 only (d) 1, 2 and 3

Answer: (a) Explanation: A river delta is a landform that forms from deposition of sediment carried by a river as the flow leaves its mouth and enters slower-moving or stagnant water. Statement 1 and 2 are correct. The necessary conditions for delta formation are:

Sea adjoining the delta should be shallow.

The coast should be sheltered and preferably tide-less.

No strong currents running at right angles to the river mouths. Hence, statement 3 is incorrect.

Source: G C Leong - Certificate Human and Physical Geography, Chapter 5, Page 44 50. There are two places A and B. A is located on 50°E longitude while B is

located on 10°W longitude. If winds blow with equal speed at these two places then

Page 41: Answer Key & Exp for GS Prelim Test-3 held on 2nd Dec’ 2018 · Commission report 1930, the 3 Round Table Conferences and the White Paper on Constitutional Reforms. What is called

Page 41 of 74

(a) relatively more Coriolis force will be experienced by wind at place A (b) relatively more Coriolis force will be experienced by wind at place B (c) wind at both places A and B will experience same Coriolis force (d) cannot be determined

Answer: (d) Explanation: Coriolis force is an apparent force caused by the earth’s rotation. The Coriolis force is responsible for deflecting winds towards the right in the northern hemisphere and towards the left in the southern hemisphere. This is also known as ‘Ferrel’s Law’. The Coriolis force is absent at the equator but increases progressively towards the poles. The magnitude of Coriolis force depends on the speed of wind and latitude (and not longitude) where the wind blows. The information about latitude is not given in the question text. Hence, it cannot be determined that wind at which place will experience more Coriolis force. Source: Geography NCERT Class 10th, Page 28 51. Which of the pairs given below is/are correctly matched?

List-I List-II

1. Hamada : Rocky desert

2. Eskers : Oval elongated hummocks composed of boulder clay

3. Kettle lakes : Small depressions formed by wind

4. Yardangs : Fluvio-glacial deposit by terminal moraines

Select the correct answer using the code given below: (a) 1 only (b) 2 only (c) 3 and 4 only (d) 1, 2, 3 and 4

Answer: (a) Explanation: Wind, Glaciers, rivers, waves etc all form different kinds of landforms. The landforms formed by glaciers are drumlins, eskers, kettle lakes, etc. Pair 2 is incorrect. Drumlins: Oval elongated hummocks composed of boulder clay Eskers: Long narrow and sinuous ridges formed by sub-glacial streams Pair 3 is incorrect. Kettle lakes: Depressions formed in outwash plains The landforms formed by wind are mushroom rocks, yardangs, hamada etc: Mushroom rocks: Rock Pillars eroded near the bases Pair 4 is incorrect. Yardangs: Vertical bands aligned in the direction of prevailing winds.

Page 42: Answer Key & Exp for GS Prelim Test-3 held on 2nd Dec’ 2018 · Commission report 1930, the 3 Round Table Conferences and the White Paper on Constitutional Reforms. What is called

Page 42 of 74

Pair 1 is correct .Hamada: A type of desert landscape consisting of high, largely barren, hard rocky plateaus, where most of the sand has been removed by deflation Source: G C Leong - Certificate Human and Physical Geography - Chapters 6 and 7, Page no. 48-60 52. Which among the following has become the ‘first fully organic’ State of

India? (a) Himachal Pradesh (b) Sikkim (c) Jammu and Kashmir (d) Arunachal Pradesh

Answer: (b) Explanation: Organic farming is an agricultural system that works in harmony with nature. It largely excludes the use of synthetic inputs (such as fertilizers, pesticides, hormones, feed additives etc.) and relies upon crop rotation, crop residues, animal manures, off-farm organic waste, mineral grade rock additives and biological system of nutrient mobilization and plant protection. Sikkim became India’s first fully organic state in 2016. Source:https://www.downtoearth.org.in/news/agriculture/india-has-the-highest-number-of-organic-farmers-globally-but-most-of-them-are-struggling-61289 53. Carefully observe the following map:

Which one of the following crops is grown in the shaded (grey colour) areas as shown in the India map?

(a) Wheat (b) Cotton (c) Sugarcane (d) Jute

Answer: (c) Explanation: Sugarcane is a crop of tropical regions. Under rainfed conditions, it is cultivated in sub-humid and humid climates. But it is largely an irrigated crop in India.

Page 43: Answer Key & Exp for GS Prelim Test-3 held on 2nd Dec’ 2018 · Commission report 1930, the 3 Round Table Conferences and the White Paper on Constitutional Reforms. What is called

Page 43 of 74

In Indo-Gangetic plain, its cultivation is largely concentrated in Uttar Pradesh.

Sugarcane growing area in western India is spread over Maharashtra and Gujarat.

In southern India, it is cultivated in irrigated tracts of Karnataka, Tamil Nadu and Andhra Pradesh.

India is the second largest producer of sugarcane after Brazil. It accounts for about 23 per cent of the world production of sugarcane. But it occupies only 2.4 per cent of total cropped area in the country. Uttar Pradesh produces about two-fifth of sugarcane of the country. But its yield is low in northern India. Maharashtra, Karnataka, Tamil Nadu and Andhra Pradesh are other leading producers of this crop where yield level of sugarcane is high. Source: NCERT – India People and Economy Class XII: Chapter - 5, Page No 52 54. Consider the following statements in the context of ‘Millets’:

(1) Maharashtra is the largest producer of jowar. (2) The year 2018 has been declared as the year of millets by the Indian

government. Which of the statements given above is/are correct? (a) 1 only (b) 2 only (c) Both 1 and 2 (d) Neither 1 nor 2

Answer: (c) Explanation: Jowar, bajra and ragi are the important millets grown in India. Though, these are known as coarse grains, they have very high nutritional value. For example, ragi is very rich in iron, calcium, and other micro nutrients.

Jowar is the third most important food crop with respect to area and production. It is a rain-fed crop mostly grown in the moist areas which hardly needs irrigation.

Statement 1 is correct. Maharashtra is the largest producer of jowar followed by Karnataka, Andhra Pradesh and Madhya Pradesh.

Bajra grows well on sandy soils and shallow black soil. Rajasthan is the largest producer of bajra followed by Uttar Pradesh, Maharashtra, Gujarat and Haryana.

Ragi is a crop of dry regions and grows well on red, black, sandy, loamy and shallow black soils. Karnataka is the largest producer of ragi followed by Tamil Nadu.

Statement 2 is correct. The government has declared 2018 as 'National Year of Millets’ to boost the cultivation of millets like ragi and jowar on a mission mode to achieve nutritional security.

Page 44: Answer Key & Exp for GS Prelim Test-3 held on 2nd Dec’ 2018 · Commission report 1930, the 3 Round Table Conferences and the White Paper on Constitutional Reforms. What is called

Page 44 of 74

Source: NCERT India People and Economy Class XII: Chapter - 5, Page No 47 55. A farmer cultivates a small plot of land using simple tools and more use of

labour. A climate having large number of days with sunshine and fertile soils permits growing of more than one crop annually on the same plot. Rice is the main crop. Other crops include wheat, maize, pulses and oilseeds. It is prevalent in the thickly populated areas of the monsoon regions of south, southeast and east Asia. The description is of: (a) Shifting cultivation (b) Intensive subsistence farming (c) Commercial farming (d) Vertical farming

Answer: (b) Explanation: Option (b) is correct. The description provided in question text is for ‘Intensive subsistence farming’. Shifting cultivation: Shifting cultivation is practised in the thickly forested areas of Amazon basin, tropical Africa, parts of south-east Asia and north-east India. These are the areas of heavy rainfall and quick regeneration of vegetation. A plot of land is cleared by felling the trees and burning them. The ashes are then mixed with the soil. Crops like maize, yam, potatoes and cassava are grown. After the soil loses its fertility, the land is abandoned and the cultivator moves to a new plot. Shifting cultivation is also known as ‘slash and burn’ agriculture. Commercial farming: In commercial farming, crops are grown and animals are reared for sale in market. The area cultivated and the amount of capital used is large. Most of the work is done by machines. Commercial farming includes commercial grain farming, mixed farming and plantation agriculture. Vertical farming: It is the practice of producing food and medicine in vertically stacked layers, vertically inclined surfaces and/or integrated in other structures (such as in a skyscraper, used warehouse, or shipping container). Source: NCERT - Resources and Development, Class VIII: Chapter- 4, Page No 42 56. Which of the following types of cultivation was developed by ‘European’

colonists? (a) Viticulture (b) Kolkoz (c) Mixed farming (d) Plantation

Answer: (d)

Page 45: Answer Key & Exp for GS Prelim Test-3 held on 2nd Dec’ 2018 · Commission report 1930, the 3 Round Table Conferences and the White Paper on Constitutional Reforms. What is called

Page 45 of 74

Explanation: Plantation: Plantation agriculture was introduced by the Europeans in colonies situated in the tropics. Some of the important plantation crops are indigo (during 19th century), tea, coffee, cocoa, rubber, cotton, oil palm, sugarcane, bananas and pineapples. The characteristic features of this type of farming are large estates or plantations, large capital investment, managerial and technical support, scientific methods of cultivation, single crop specialisation, cheap labour, and a good system of transportation which links the estates to the factories and markets for the export of the products. Viticulture: Viticulture or grape cultivation is a speciality of the Mediterranean region. Best quality wines in the world with distinctive flavours are produced from high quality grapes in various countries of this region. The inferior grapes are dried into raisins and currants. Collective farming: The basic principle behind this type of farming is based on social ownership of the means of production and collective labour. Collective farming or the model of Kolkhoz was introduced in erstwhile Soviet Union to improve upon the inefficiency of the previous methods of agriculture and to boost agricultural production for self-sufficiency. Mixed farming: A system of farming which involves the growing of crops as well as the raising of livestock. This form of agriculture is found in the highly developed parts of the world, e.g. North-western Europe, Eastern North America, parts of Eurasia and the temperate latitudes of Southern continents. Source: NCERT - Fundamentals of Human Geography, Class XII: Page No 37 57. What do you mean by the term 'Truck Farming’?

(a) Farming wherein the farmers specialise in growing vegetables only (b) Farming of fruits and flowers only (c) Soil-less farming in a truck (d) Abundant production of large carriers like trucks, containers, etc

Answer: (a) Explanation: Truck farming is a horticultural practice of growing one or more vegetable crops on a large scale for shipment to distant markets. It is usually less intensive and diversified than market gardening. The major truck-farming areas are in California, Texas, Florida, along the Atlantic Coastal Plain, and in the Great Lakes area. Among the most important truck crops are tomatoes, lettuce, melons, beets, broccoli, celery, radishes, onions, cabbage, and strawberries which are perishable in nature. Source: NCERT - Fundamentals of human geography, Std 12th: Chapter - 5, Page No 40

Page 46: Answer Key & Exp for GS Prelim Test-3 held on 2nd Dec’ 2018 · Commission report 1930, the 3 Round Table Conferences and the White Paper on Constitutional Reforms. What is called

Page 46 of 74

58. The asteroid belt is found between ____________.

(a) Neptune and Pluto (b) Mars and Jupiter (c) Venus and earth (d) Saturn and Jupiter

Answer: (b) Explanation: There are numerous tiny bodies that move around the sun apart from stars, planets and satellites which are called as asteroids. They are found between the orbits of mars and Jupiter. Our solar system is filled with a wide assortment of celestial bodies - the Sun, our eight planets, dwarf planets, and asteroids. Asteroids are small bodies that are believed to be left over from the beginning of the solar system 4.6 billion years ago. They are rocky objects with round or irregular shapes up to several hundred km across, but most are much smaller. More than 100,000 asteroids lie in a belt between Mars and Jupiter. Scientists think that this debris may be the remains of an early planet, which broke up early in the solar system. Several thousand of the largest asteroids in this belt have been given names. Source: NCERT Class 6, chapter 1, Page 6 59. Consider the following pairs:

Rainfall Type Feature

1. Convectional rain : It is common in polar regions

2. Orographic rain : An air mass comes across a mountain, ascends and cools down to precipitate

3. Cyclonic rain : Convergence of two different air masses

Which of the pairs given above is/are incorrectly matched? (a) 1 only (b) 3 only (c) 1 and 2 only (d) 1 and 3 only

Answer: (a) Explanation: Pair 1 is incorrectly matched: Convectional rainfall is most common in regions that are intensely heated, either during day, as in tropics, or in summer as in temperate interior. When the earth’s surface is heated by conduction, moisture-laden vapour rises because heated air always expands and becomes light and rises. It loses heat and consequently, condensation takes place and cumulous clouds are formed. With

Page 47: Answer Key & Exp for GS Prelim Test-3 held on 2nd Dec’ 2018 · Commission report 1930, the 3 Round Table Conferences and the White Paper on Constitutional Reforms. What is called

Page 47 of 74

thunder and lightning, heavy rainfall takes place but this does not last long. Such type of rain is common in the summer or in the hotter part of the day. It is very common in the equatorial regions and interior parts of the continents, particularly in the northern hemisphere. Pair 2 is correctly matched: When a saturated air mass comes across a mountain, it is forced to ascend and as it rises, it expands; the temperature falls, and the moisture is condensed. The chief characteristic of this sort of rain is that the windward slopes receive greater rainfall. Pair 3 is correctly matched: Frontal (or Cyclonic) Rain is caused by cyclonic activity and it occurs along the fronts of the cyclone. It is formed when two masses of air of different temperature, humidity and density meets. Source: Fundamentals of Physical Geography (NCERT 11th) - Chapter 11, Page No.- 101 60. Which of the following research stations of India have been set up in

Antarctica? (1) Himadri (2) Bharti (3) Dakshin Gangotri (4) Maitri Select the correct answer using the code given below: (a) 1, 2 and 3 only (b) 1, 2 and 4 only (c) 1, 3 and 4 only (d) 2, 3 and 4 only

Answer: (d) Explanation: Antarctica is Earth's southernmost continent. It contains the geographic South Pole and is situated in the Antarctic region of the Southern Hemisphere, almost entirely south of the Antarctic Circle, and is surrounded by the Southern Ocean. The Arctic is a polar region located at the northernmost part of Earth. The Arctic consists of the Arctic Ocean, adjacent seas, and parts of Alaska (United States), Finland, Greenland (Denmark), Iceland, Northern Canada, Norway, Russia and Sweden. Land within the Arctic region has seasonally varying snow and ice cover, with predominantly treeless permafrost (permanently frozen underground ice)-containing tundra. Arctic seas contain seasonal sea ice in many places.

Himadri is a research station of India in Arctic.

Bharti is the recent research station in Antarctica established in 2012.

Dakshin Gangotri was the first scientific base station of India situated in Antarctica, part of the Indian Antarctic Programme. It is located at a distance of 2,500 kms (1,600 mi) from the South Pole. It is currently being used as a supply base and transit camp. It was succeeded by the Maitri, which was set up in a moderate climatic zone at a distance of 90 km and made operational in 1990.

Page 48: Answer Key & Exp for GS Prelim Test-3 held on 2nd Dec’ 2018 · Commission report 1930, the 3 Round Table Conferences and the White Paper on Constitutional Reforms. What is called

Page 48 of 74

Source: NCERT Class 6, Chapter 5, Page 33 61. Which of the following is an example of ‘block mountain’?

(a) Ural Mountains (b) Appalachians (c) Vosges Mountains (d) Mt. Kilimanjaro

Answer: (c) Explanation: Ural Mountains and Appalachians are old fold mountains found in Russia and North America respectively. They have low elevation. Mt. Kilimanjaro in Africa is formed due to volcanic activity. Vosges Mountain in Europe is an example of Block Mountains. Block Mountains: These are created when large areas or blocks of earth are broken and displaced vertically. The uplifted blocks are termed as horsts and the lowered blocks are called graben. The Great African Rift Valley (valley floor is graben), The Rhine Valley and the Vosges Mountain in Europe are examples. In India, the Vindhyas and Satpuras are examples of block mountains. Block Mountains are also called fault Block Mountains since they are formed due to faulting as a result of tensile and compressive forces. There are two basic types.

Tilted block mountains have one steep side contrasted by a gentle slope on the other side.

Lifted block mountains have a flat top and extremely steep slopes.

Source: NCERT Class 6, Chapter 6, Page 41 62. Which of the following are basic factors controlling formation of soil?

(1) Parent Material

Page 49: Answer Key & Exp for GS Prelim Test-3 held on 2nd Dec’ 2018 · Commission report 1930, the 3 Round Table Conferences and the White Paper on Constitutional Reforms. What is called

Page 49 of 74

(2) Topography (3) Climate (4) Biological Activity (5) Time Select the correct answer using the code given below: (a) 1, 2, 3 and 4 only (b) 1, 3, 4 and 5 only (c) 2, 3, 4 and 5 only (d) 1, 2, 3, 4 and 5

Answer: (d) Explanation: Pedogenesis is the process of soil development. Various factors which influence this process are: Parent material is a passive control factor in soil formation whose texture and structure determine the type of soil. Topography, a passive factor determines the amount of exposure of parent material surface. Topography has a strong influence on soil development. Soils on the side of hills tend to be shallow, due to erosional losses. Soils on the tops of hills tend to be deep, but lighter in colour, due to downward leaching losses. Soils in the valleys tend to be deeper, darker, and contain more horizons. This is due to increased material deposition from hillside erosion, material accumulation from downward leaching from the tops of hills, and the collection of greater quantities of water in the low lying areas. Climate, biological activity and time are the active factors in soil formation which play role in soil development, addition of organic matter and the soil forming process respectively. Soil organisms play a vital role in the degradation of organic matter and subsequent soil humus formation. When plants die, leaves are dropped onto the soil surface where microorganisms can decay plant tissue. The organic matter is used as an energy source for microorganisms, increasing their population in the soil. These organisms utilize easily digestible materials (like simple sugars and carbohydrates) found in the plant material, leaving more resistant materials (such as fats and waxes) behind. The material left behind is not easily decomposed; it comprises the humus found in soil. Humus acts as a gluing agent, essentially holding primary soil particles (sand, silt, clay) together to form secondary aggregates. These organisms and the humus they help create aid in the soil development and the formation of soil horizons. Source: NCERT Class 11, Chapter 6, Page 55 63. ‘Duncan passage' is located between which two islands?

(a) Little Nicobar and Great Nicobar (b) South Andaman and Little Andaman (c) Car Nicobar and Little Nicobar (d) None of the above

Page 50: Answer Key & Exp for GS Prelim Test-3 held on 2nd Dec’ 2018 · Commission report 1930, the 3 Round Table Conferences and the White Paper on Constitutional Reforms. What is called

Page 50 of 74

Answer: (b) Explanation:

Duncan passage is located between South Andaman and Little Andaman.

Pryce channel exists between Little Nicobar and Great Nicobar.

Sombrero Channel and Kimios Bay exist between Car Nicobar and Little Nicobar.

Ten Degree Channel lies between Andaman and Nicobar.

Source: NCERT class 11th, Chapter 1, Page 3 64. The Tropic of Cancer does not pass through which of the following States?

(a) Rajasthan (b) Chhattisgarh (c) Tripura (d) Manipur

Page 51: Answer Key & Exp for GS Prelim Test-3 held on 2nd Dec’ 2018 · Commission report 1930, the 3 Round Table Conferences and the White Paper on Constitutional Reforms. What is called

Page 51 of 74

Answer: (d) Explanation: The Tropic of Cancer is the circle marking the latitude 23.5 degrees north, where the sun is directly overhead at noon on June 21, the beginning of summer in the northern hemisphere. Tropic of cancer passes through the following Indian states (in order from east to west): 1. Mizoram 2. Tripura 3. West Bengal 4. Jharkhand 5. Chhattisgarh 6. Madhya Pradesh 7. Rajasthan 8. Gujarat

Source: NCERT class 11th, Chapter 1, Page 3 65. Consider the following national parks:

(1) Jim Corbett National Park (2) Dudhwa National Park (3) Keoladeo Ghana National Park (4) Simlipal National Park (5) Kanha National Park Select the correct order (from north to south) of national parks using the code given below: (a) 1, 2, 3, 4, 5

Page 52: Answer Key & Exp for GS Prelim Test-3 held on 2nd Dec’ 2018 · Commission report 1930, the 3 Round Table Conferences and the White Paper on Constitutional Reforms. What is called

Page 52 of 74

(b) 1, 2, 3, 5, 4 (c) 1, 3, 2, 4, 5 (d) 2, 3, 4, 5, 1

Answer: (b) Explanation: Jim Corbett National Park – Uttarakhand: Jim Corbett National Park is the oldest national park in India and was established in 1936 as Hailey National Park to protect the endangered Bengal tiger. It is a part of the larger Corbett Tiger Reserve. It lies in the Nainital district of Uttarakhand. Dudhwa National Park – Uttar Pradesh: The Dudhwa National Park in Lakhimpur-Kheri District of Uttar Pradesh, adjacent to Nepal border, is one of the major projects for wildlife preservation in India. Keoladeo Ghana National Park – Rajasthan: Keoladeo Ghana National Park formerly known as the Bharatpur Bird Sanctuary in Bharatpur, Rajasthan, India is a famous avifauna sanctuary that hosts thousands of birds, especially during the winter season. Over 230 species of birds are known to be resident. It was declared a protected sanctuary in 1971. It is also a World Heritage Site. Simlipal National Park – Odisha: Simlipal National Park is a tiger reserve in the Mayurbhanj district in the Indian state of Odisha. It is part of the Similipal-Kuldiha-Hadgarh Elephant Reserve popularly known as Mayurbhanj Elephant Reserve, which includes three protected areas — Similipal Tiger Reserve, Hadgarh Wildlife Sanctuary and Kuldiha Wildlife Sanctuary Kanha National Park – Madhya Pradesh: Kanha National Park is one of the tiger reserves of India and the largest national park of Madhya Pradesh. Its area is divided into two sanctuaries, Hallon and Banjar. Source: NCERT Class 9, Chapter 5, Page 49. 66. Coral reefs require certain specific conditions to survive in the best manner.

Which of the following is correct with regard to these conditions? (a) Water temperature must be below 10°C. (b) Shallow water is not favourable. (c) They will not survive where there are cold currents. (d) Clear, oxygenated water hamper their development.

Answer: (c) Explanation: A coral reef is an underwater ecosystem characterized by reef-building corals. Reefs are formed of colonies of coral polyps held together by calcium carbonate. Most coral reefs are built from stony corals, whose polyps cluster in groups. Option (a) is incorrect. Corals generally live in water temperatures of 20–32° C. Option (b) is incorrect. Shallow water is favourable as at greater depths sunlight is too faint for photosynthesis. Option (c) is correct. They do not survive where there are cold currents as there will be upwelling of the cold water from the depths.

Page 53: Answer Key & Exp for GS Prelim Test-3 held on 2nd Dec’ 2018 · Commission report 1930, the 3 Round Table Conferences and the White Paper on Constitutional Reforms. What is called

Page 53 of 74

Option (d) is incorrect. Clear, oxygenated water promote their development. Source: G C Leong Certificate Human and Physical Geography, Chapter 11 Page 80 67. The terms ‘Titli’ and ‘Luban’ have been in the news recently. What are

these? (a) Titli is a genetically modified rice variety whereas Luban is a

genetically modified cotton variety. (b) Titli is a genetically modified cotton variety whereas Luban is a

genetically modified rice variety. (c) Titli is a cyclonic storm originating in the Bay of Bengal whereas

Luban is a cyclonic storm originating in the Arabian Sea. (d) Titli is a cyclonic storm originating in the Arabian Sea whereas Luban

is a cyclonic storm originating in the Bay of Bengal. Answer: (c) Explanation: The India Meteorological Department (IMD) called the formation of the two very severe cyclones—Titli (over the Bay of Bengal) and Luban (over the Arabian Sea)—on two sides of the Indian mainland as ‘rarest of rare’ occurrences. The IMD also said the movement of both these storms was unique. While Titli changed its direction and moved towards the northeast after making a landfall, Luban too kept going in different directions over the 9 days that it travelled through the south-eastern Arabian Sea towards Yemen and Oman on the Gulf coast and then made landfall on October 13. Source: https://www.downtoearth.org.in/news/natural-disasters/imd-calls-titli-luban-cyclones-rarest-of-rare-occurrences-61900 68. Consider the following statements about ‘Iron ore’:

(1) China is the largest producer of iron ore in the world. (2) Hematite, Magnetite, Limonite and Siderite are the four varieties of

iron ore. (3) Jharkhand is one of the major iron ore producing state in India. Which of the statements given above is/are correct? (a) 1 only (b) 1 and 2 only (c) 1 and 3 only (d) 1, 2 and 3

Answer: (d) Explanation: Statement (a) is correct. China is the largest producer of iron ore in the world. Most of it is consumed within country only for domestic use. Statement (b) is correct. There are four varieties of iron ore - Hematite, Magnetite, Limonite and Siderite.

Page 54: Answer Key & Exp for GS Prelim Test-3 held on 2nd Dec’ 2018 · Commission report 1930, the 3 Round Table Conferences and the White Paper on Constitutional Reforms. What is called

Page 54 of 74

Statement (c) is correct. The major iron ore producing states in India are Jharkhand, Karnataka, Odisha and Chhattisgarh. Goa, Maharashtra, Andhra Pradesh and Tamil Nadu are minor producers. The main exporting nations are Brazil, Australia, Canada and India. These four countries together account for about 70% of total export of iron ore. Source: Yellow Book - Geography Natural Resources- Chapter- 2, Page no 196-198 69. Match List-I with List-II and select the correct answer using the code given

below the lists:

List-I (Resource)

List-II (Location)

A. Lead 1. Jharia

B. Coal 2. Zawar Mines

C. Petroleum Oil 3. Sambalpur

D. Bauxite 4. Digboi

Codes A B C D (a) 2 1 4 3 (b) 3 4 2 1 (c) 3 4 1 2 (d) 2 1 3 4

Answer: (a) Explanation: In India, lead production comes from the Zawar mines of Rajasthan and Arcot district of Tamil Nadu. Coal lie in Jharkhand-Bengal coal belt and the important coal fields in this region are Raniganj, Jharia, Bokaro, Giridih and Karanpura. The major oil producing areas are Digboi, Ankaleshwar, Mumbai high, Kalol. Odisha happens to be the largest producer of bauxite. Sambalpur and Kalahandi are the leading producers of bauxite. Source: Yellow Book - ‘Concepts of Geography and Natural Resources’ - Chapter-1 and 2, Page no 185 - 200 70. With reference to ‘Nuclear Energy Resources’, consider the following

statements: (1) Thorium is mainly obtained from monazite and ilmenite. (2) World’s richest monazite deposits occur in Maharashtra. Which of the statements given above is/are incorrect? (a) 1 only (b) 2 only (c) Both 1 and 2

Page 55: Answer Key & Exp for GS Prelim Test-3 held on 2nd Dec’ 2018 · Commission report 1930, the 3 Round Table Conferences and the White Paper on Constitutional Reforms. What is called

Page 55 of 74

(d) Neither 1 nor 2 Answer: (b) Explanation: Statement (1) is correct. Thorium is mainly obtained from monazite and ilmenite in the beach sands along the coast of Kerala and Tamil Nadu. Statement (2) is incorrect. World’s richest monazite deposits occur in Palakkad and Kollam districts of Kerala, Vishakhapatnam in Andhra Pradesh and Mahanadi river delta of Odisha. Source: NCERT Class XII - India People and Economy – Chapter- 7, Page no 82 71. Match the ‘Oil Refineries’ marked in the map with their location as provided

below: Locations: (1) Guwahati (2) Barauni (3) Mathura (4) Jamnagar

Select the correct answer using the code below:

Codes A B C D

(a) 4 2 3 1 (b) 3 4 2 1 (c) 3 4 1 2 (d) 4 3 2 1 Answer: (d) Explanation: Oil refinery or petroleum refinery is an industrial process plant where

crude oil is transformed and refined into more useful products such as petroleum naphtha, gasoline, diesel fuel, asphalt base, heating oil, kerosene, liquefied petroleum gas, jet fuel and fuel oils. A: Jamnagar, Gujarat B: Mathura, Uttar Pradesh C: Barauni, Bihar D: Guwahati, Assam Source: NCERT India People and Economy – Class XII – Chapter - 7, Page no 81

Page 56: Answer Key & Exp for GS Prelim Test-3 held on 2nd Dec’ 2018 · Commission report 1930, the 3 Round Table Conferences and the White Paper on Constitutional Reforms. What is called

Page 56 of 74

72. Which of the following does not belong to the list of leading sugarcane-producing States? (a) Uttar Pradesh (b) Andhra Pradesh (c) Madhya Pradesh (d) Maharashtra

Answer (c) Explanation: The leading sugarcane-producing States are the leading sugar producers as well. They are Uttar Pradesh, Maharashtra, Andhra Pradesh, Tamil Nadu, Karnataka, Bihar and Jharkhand. There are more than 250 sugar mills in India, with the largest number located in Uttar Pradesh followed by Maharashtra. Source: NCERT – India People and Economy - Class XII – Chapter - 8, page no 94 73. Consider the following statements:

Assertion: The western coastal plains are narrower than eastern coastal plains. Reason: The west coast is a coast of submergence and the east coast is a coast of emergence. Select the correct answer using the code given below: (a) Both Assertion and Reason are true and Reason is the correct

explanation of Assertion. (b) Both Assertion and Reason are true but Reason is not the correct

explanation of Assertion. (c) Assertion is true but Reason is wrong. (d) Assertion is wrong but Reason is true

Answer: (a) Explanation: The western coastal plains are an example of submerged coastal plain. Because of this submergence, it is a narrow belt and provides natural conditions for the development of ports and harbours. As compared to the western coastal plain, the eastern coastal plain is broader because it is an example of an emergent coast. The submergence and emergence of the coasts can be attributed to the tilting of peninsula due to Himalayan orogeny. Source: NCERT, India: Physical Environment XI Class, Chapter 2, Page 18. 74. ‘Namcha Barwa’ mountain peak is situated in ____________?

(a) Arunachal Himalayas (b) Darjeeling and Sikkim Himalayas (c) Himachal and Uttarakhand Himalayas (d) Kashmir and Northwestern Himalayas

Page 57: Answer Key & Exp for GS Prelim Test-3 held on 2nd Dec’ 2018 · Commission report 1930, the 3 Round Table Conferences and the White Paper on Constitutional Reforms. What is called

Page 57 of 74

Answer: (a) Explanation: Namcha Barwa is a part of Arunachal Himalayas. It stands inside the Great Bend of the Yarlung Tsangpo River as the river enters its notable gorge across the Himalaya. Source: NCERT, India: Physical Environment Class XI, Chapter 2, Page 13. 75. Which of the following statement correctly describes the ‘Kayals’?

(a) Relict hills in the scarped plateaus north of Vindhya range. (b) Backwaters or brackish lagoons of Kerala which are used for fishing,

inland navigation and tourism. (c) Shifting sand dunes in the Great Indian Desert in Western Rajasthan. (d) Salty marshes in Great Runn of Kutch in Northern Gujarat.

Answer: (b) Explanation: The Kerala backwaters are a chain of brackish lagoons and lakes lying parallel to the Arabian Sea coast (known as the Malabar Coast) of Kerala state in southern India. The network includes five large lakes linked by canals, both man-made and natural. The backwaters were formed by the action of waves and shore currents creating low barrier islands across the mouths of the many rivers flowing down from the Western Ghats range. The 'kayals' are used for fishing, inland navigation and tourism (every year the famous Nehru Trophy Vallamkali - boat race is held in Punnamada kayal in Kerala). Source: NCERT, India: Physical Environment, Class XI Chapter 2, Page 18. 76. Which of the following is/are the minor plates according to ‘Plate Tectonic

Theory’? (1) Cocos plate (2) Antarctica plate (3) Caroline plate (4) Arabian plate Select the correct answer using the code given below: (a) 2 only (b) 1 and 4 only (c) 1, 3 and 4 only (d) 1, 2, 3 and 4

Answer: (c) Explanation: According to Plate Tectonic theory, a tectonic plate (also called lithospheric plate) is a massive, irregularly-shaped slab of solid rock, generally composed of both continental and oceanic lithosphere. Plates move horizontally over the asthenosphere as rigid units. A plate may be referred to as the

Page 58: Answer Key & Exp for GS Prelim Test-3 held on 2nd Dec’ 2018 · Commission report 1930, the 3 Round Table Conferences and the White Paper on Constitutional Reforms. What is called

Page 58 of 74

continental plate or oceanic plate depending on which of the two occupy a larger portion of the plate. The theory of plate tectonics proposes that the earth’s lithosphere is divided into seven major and some minor plates.

The major plates are as follows: 1) Antarctica and the surrounding

oceanic plate 2) North American (with western

Atlantic floor separated from the South American plate along the Caribbean islands) plate

3) South American (with western Atlantic floor separated from the North American plate along the Caribbean islands) plate

4) Pacific plate 5) India-Australia-New Zealand plate 6) Africa with the eastern Atlantic floor

plate 7) Eurasia and the adjacent oceanic

plate.

Some important minor plates are listed below: 1) Cocos plate: Between Central

America and Pacific plate 2) Nazca plate: Between South

America and Pacific plate 3) Arabian plate: Mostly the Saudi

Arabian landmass 4) Philippine plate: Between the

Asiatic and Pacific plate 5) Caroline plate: Between the

Philippine and Indian plate (North of New Guinea)

6) Fuji plate: North-east of Australia.

Source: NCERT- Class XI Fundamental of Physical Geography – Chapter 4, Page no. 35-36 77. Consider the following pairs:

Intrusive Landforms

Feature

1. Laccolith : Dome shaped upper su rface

2. Lopolith : Lens Shaped

3. Phacolith : Saucer Shaped

Page 59: Answer Key & Exp for GS Prelim Test-3 held on 2nd Dec’ 2018 · Commission report 1930, the 3 Round Table Conferences and the White Paper on Constitutional Reforms. What is called

Page 59 of 74

Which of the pairs given above is/are correctly matched? (a) 1 only (b) 2 only (c) 1 and 2 only (d) 1 and 3 only

Answer: (a) Explanation: Igneous intrusions form when magma cools and solidifies before it reaches the surface. Three common types of intrusion are sills, dykes, and batholiths. Igneous intrusions on a larger scale are the various types of ‘ – liths’. Pair 1 is correctly matched. A laccolith is a large blister or igneous mound with a dome-shaped upper surface and level base fed by a pipe-like conduit from below. Pairs 2 and 3 are incorrectly matched. A lopolith is another variety of igneous intrusion with a saucer shape. A phacolith is a lens-shaped mass of igneous rocks occupying the crest of an anticline or the bottom of a syncline and being fed from beneath. Source: G. C. Leong - Certificate Physical and Human Geography – Chapter 3, Page 20 78. Which among the following features are best developed in the upper

course of a river: (1) River Capture (2) Rapids (3) Interlocking Spurs (4) Ox-bow lakes Select the correct answer using the code given below: (a) 1 only (b) 1 and 2 only (c) 2 and 3 only (d) 1, 3 and 4 only

Answer: (b) Explanation: Some of the outstanding features that are often best developed in the upper course of a river include river capture, rapids, cataracts and waterfalls. River Capture: The natural diversion of the headwaters of one stream into the channel of another, typically resulting from rapid headward erosion by the latter stream. Rapids: Rapids are sections of a river where the river bed has a relatively steep gradient, causing an increase in water velocity and turbulence. Rapids are hydrological features between a run (a smoothly flowing part of a stream) and a cascade.

Page 60: Answer Key & Exp for GS Prelim Test-3 held on 2nd Dec’ 2018 · Commission report 1930, the 3 Round Table Conferences and the White Paper on Constitutional Reforms. What is called

Page 60 of 74

Interlocking spurs are developed in middle course of river whereas Ox-bow lakes are formed in lower course of river. An interlocking spur, also known as an overlapping spur, is one of the many projecting ridges that extends alternately from the opposite sides of the wall of a young, V-shaped valley. An oxbow lake is a U-shaped lake that forms when a wide meander from the main stem of a river is cut off, creating a free-standing body of water. This landform is so named for its distinctive curved shape, which resembles the bow pin of an oxbow. Source: G.C. Leong- Certificate Human and Physical Geography, Chapter- 5, Page no - 41, 42, and 43. 79. With reference to ‘Hot-spot’ volcano, consider the following statements:

(1) They are created by mantle plumes. (2) The magma coming out of this kind of volcano is highly viscous. Which of the statements given above is/are correct? (a) 1 only (b) 2 only (c) Both 1 and 2 (d) Neither 1 nor 2

Answer: (a) Explanation: About 5% of volcanoes are not near the margins of tectonic plates. They are over especially hot places in the Earth’s interior called Hot Spots. Statement 1 is correct: They are created by mantle plumes/conduit -hot currents that rise all the way from the deep mantle or near mantle-core boundary. High heat and lower pressure at the base of the lithosphere (tectonic plate) facilitates melting of the rock. This melt, called magma, rises through cracks and erupts to form volcanoes. As the tectonic plate moves over the stationary hot spot, the volcanoes are rafted away and new ones form in their place. This results in chains of volcanoes, such as the Hawaiian Islands. Statement 2 is incorrect: The lava oozes out of hot-spot volcanoes is highly fluidic and thus spreads out to form shield volcanoes or plateaux such as Massif Central in France. The geysers, hot springs and bubbling mud pots of Yellowstone National Park, USA, indicate a hot-spot below.

Page 61: Answer Key & Exp for GS Prelim Test-3 held on 2nd Dec’ 2018 · Commission report 1930, the 3 Round Table Conferences and the White Paper on Constitutional Reforms. What is called

Page 61 of 74

Source: Yellow book ‘Concepts of Geography and Natural Resources’- Chapter 6, Page 47 80. Consider the following statements:

(1) The swarms of rounded hummock resulting from the deposition of glacial till are called drumlins.

(2) Zeugen is a table-shaped area of rock found in arid and semi-arid areas formed when more resistant rock is reduced at a slower rate than softer rocks around it under the effects of wind erosion.

Which of the statements given above is/are correct? (a) 1 only (b) 2 only (c) Both 1 and 2 (d) Neither 1 nor 2

Answer: (c) Explanation: Drumlins: The swarms of rounded hummock resulting from the deposition of glacial till are called drumlins. They look like inverted boat or spoon. They appear a little steeper at the onset side and taper off at the leeward side. They are arranged diagonally and so are commonly described as having a basket of eggs topography.

Zeugens: These are tabular masses (aeolian erosional landform) which have a layer of soft rocks lying beneath a surface layer of more resistant rocks. The

Page 62: Answer Key & Exp for GS Prelim Test-3 held on 2nd Dec’ 2018 · Commission report 1930, the 3 Round Table Conferences and the White Paper on Constitutional Reforms. What is called

Page 62 of 74

sculpting effects of wind abrasion wear them into a weird-looking ‘ridge and furrow landscape. Mechanical weathering initiates their formation by opening up joints of the surface rocks. Wind abrasion further eats into the underlying soft layer so that deep furrows are developed. The hard rocks then stand above the furrows as ridges or zeugen.

Source: Yellow book ‘Concepts of Geography and Natural Resources’- Chapter 9, Pages 72,73,77,78

Page 63: Answer Key & Exp for GS Prelim Test-3 held on 2nd Dec’ 2018 · Commission report 1930, the 3 Round Table Conferences and the White Paper on Constitutional Reforms. What is called

Page 63 of 74

81. Hydrazine is preferred as a propellant in the space industry because of: (1) Its high performance characteristics. (2) Its environment-friendly nature. (3) Its safe storage, ground handling and transportation benefits. Select the correct answer using the code given below: (a) 1 only (b) 1 and 3 only (c) 2 and 3 only (d) 1, 2 and 3

Answer: (a) Explanation: Due to its high performance characteristics, hydrazine has dominated the space industry as the choice of propellant for over six decades, despite its environment and health hazards and the challenges faced in its manufacturing, storage, ground handling and transportation. Hydrazine is highly corrosive and it is also carcinogenic and toxic. To replace Hydrazine, ISRO is working on the HAN (Hydroxyl ammonium nitrate or hydroxylamine nitrate)-based monopropellant and carrying out a variety of tests to investigate its characteristics, like thermal and catalytic decomposition and compatibility with different materials. Hence, correct answer is (a). Source: Vajiram & Ravi Institute Current Affairs Magazine: May 2018; Section:Science & Technology; Page no. 91 82. Consider the following statements:

(1) Western freight corridor runs from Ghaziabad to Jawaharlal Nehru Port Trust in Mumbai.

(2) Eastern freight corridor runs from Ludhiana in Punjab to Dankuni in West Bengal.

(3) Western corridor is being fully funded by the Asian Development Bank.

(4) Eastern corridor is being partially funded by the World Bank. Which of the statements given above is/are correct? (a) 1 and 3 only (b) 1, 2 and 3 only (c) 2 and 4 only (d) 1, 2, 3 and 4

Answer: (c) Explanation:

Statement 1 is incorrect. The 1500 km western freight corridor runs from Dadri near Delhi to Jawaharlal Nehru Port Trust in Mumbai.

Statement 2 is correct. The 1800-km eastern corridor is from Ludhiana in Punjab to Dankuni in West Bengal.

Page 64: Answer Key & Exp for GS Prelim Test-3 held on 2nd Dec’ 2018 · Commission report 1930, the 3 Round Table Conferences and the White Paper on Constitutional Reforms. What is called

Page 64 of 74

Statement 3 is incorrect. The construction of the western corridor is being fully funded by the Japanese International Cooperation Agency, which has provided around Rs 33000 crore as soft loan.

Statement 2 is correct. The eastern freight corridor is being partially funded by the World Bank.

Source: Vajiram & Ravi Institute Current Affairs Magazine: May 2018; Section: Economy; Page no. 82 83. With reference to the ‘Indus Water Treaty’, which of the following

statements is/are correct? (1) It gave India control over the Indus, the Jhelum and the Chenab

rivers. (2) It gave Pakistan exclusive right to use waters from the Beas, the Ravi

and the Sutlej rivers (3) The Pakal Dul Hydroelectric Power Project is proposed on the

Marusadar river, a tributary of Chenab river, in Jammu and Kashmir. Select the correct answer using the code given below: (a) 1 and 2 only (b) 1 and 3 only (c) 3 only (d) 1, 2 and 3

Answer: (c) Explanation: Statements 1 and 2 are incorrect: The water-distribution treaty between India and Pakistan known as Indus Water Treaty (IWT) was signed in on September 19, 1960, gave Pakistan the exclusive right to use waters from the Indus and its westward flowing tributaries, the Jhelum and the Chenab. However, Jammu & Kashmir was allowed to use some water from these rivers for hydropower generation, domestic use and agriculture. The IWT gave India the control over three eastern rivers— the Beas, the Ravi and the Sutlej. Since Pakistan's rivers flow through India first, the treaty allowed India to use them for irrigation, transport and power generation. Statement 3 is correct: Pakal Dul Hydroelectric Power Project is proposed on Marusadar River, a tributary of Chenab River, in Jammu and Kashmir. Source: Vajiram & Ravi Institute Current Affairs Magazine: May 2018; Section: International Relations; Page No. 36 84. The Government of India has decided to take the ordinance route to

implement the proposed ‘Indian Arbitration Council Act, 2017’. In this context, consider the following statements: (1) The Institution will conduct only international arbitration. (2) The Minister of Law and Justice will chair the Arbitration Council. (3) The Council will take over the undertakings of International Centre for

Alternative Dispute Resolution International Centre (ICADR). Which of the statements given above is/are correct? (a) 3 only

Page 65: Answer Key & Exp for GS Prelim Test-3 held on 2nd Dec’ 2018 · Commission report 1930, the 3 Round Table Conferences and the White Paper on Constitutional Reforms. What is called

Page 65 of 74

(b) 1 and 2 only (c) 1 and 3 only (d) 1, 2 and 3

Answer: (a) Explanation: Statement 1 is incorrect: The proposed Act aims to establish a flagship institution namely Indian Arbitration Council to conduct both international and domestic arbitration. Statement 2 is incorrect: A retired Supreme Court or High Court judge or an eminent person with special knowledge and experience in the conduct or administration of arbitration will chair the arbitration council. The person will be appointed by the central government in consultation with the Chief Justice of India. Statement 3 is correct: The council will take over the undertakings of International Centre for Alternative Dispute Resolution International Centre (ICADR). Source: Vajiram & Ravi Institute Current Affairs Magazine: May 2018; Section: Polity; Page No. 12 85. Which one of the following is the best description of ‘InSight’ that was in the

news recently? (a) It is the first-ever mission to study the deep interior of Mars. (b) It is a new satellite launched by ISRO for keeping an eye on enemies

both on land and at sea. (c) It is the first-ever mission to study the presence of water reserves on

the Moon. (d) It is an Unmanned Aerial Vehicle developed by DRDO.

Answer: (a) Explanation: InSight is a NASA Discovery Program mission that will place a single geophysical lander on Mars to study its deep interior. But InSight is more than a Mars mission - it is a terrestrial planet explorer that will address one of the most fundamental issues of planetary and solar system science - understanding the processes that shaped the rocky planets of the inner solar system (including Earth) more than four billion years ago. Source: Vajiram & Ravi Institute Current Affairs Magazine: May 2018; Section: Science & Technology; Page no. 92 86. With reference to BharatNet, consider the following statements:

(1) It aims to connect all Gram Panchayats with optical fibre by March, 2019.

(2) It proposes connecting all public institutions and every home in a village through internet.

Which of the statements given above is/are correct? (a) 1 only

Page 66: Answer Key & Exp for GS Prelim Test-3 held on 2nd Dec’ 2018 · Commission report 1930, the 3 Round Table Conferences and the White Paper on Constitutional Reforms. What is called

Page 66 of 74

(b) 2 only (c) Both 1 and 2 (d) Neither 1 nor 2

Answer: (b) Explanation: Statement 1 is incorrect: Bharat Net Project is the new brand name of National Optical Fibre Network (NOFN) which was launched in October, 2011. It was renamed BharatNet in 2015. Statement 2 is correct: BharatNet envisages delivering broadband connectivity to 250000 gram panchayats (village councils) by March, 2019.) and the Centre is now deliberating that the project should not stop there but also provide internet connection to every village home. Phases of BharatNet Phase I: The target of completing 1,00,000 GPs under phase-I of BharatNet was achieved in December 2017. Phase ll: BharatNet Phase-II is planned to connect the remaining 1,50,000 GPs, using an optimal mix of media, by 31 Mar, 2019. Phase II is being implemented through three models – state-led model, CPSU model, and private sector model. Source: Vajiram & Ravi Institute Current Affairs Magazine: May 2018; Section: Governance; Page No. 19 87. With reference to the 'Missile Technology Control Regime (MTCR)', which

of the following statements is/are correct? (1) MTCR is an informal political understanding among States that seek

to limit the proliferation of missiles and missile technology. (2) The regime was formed in 1987 by the G-7 industrialized countries. (3) India, Japan, Republic of Korea and China are the only Asian

countries who are members of the MTCR. (4) The aim of the MTCR is to restrict the proliferation of missiles and

related technology for those systems capable of carrying a 5000 kilogram payload at least 3000 kilometres.

Select the correct answer using the code given below: (a) 1 and 2 only (b) 2 and 4 only (c) 1, 2 and 4 only (d) 1, 2, 3 and 4

Answer: (a) Explanation: Statement 1 is correct. MTCR is an informal political understanding among states that seek to limit the proliferation of missiles and missile technology. Statement 2 is correct. MTCR was established by G-7 countries in 1987. Statement 3 is incorrect. China is not a member of MTCR. Statement 4 is incorrect The aim of the MTCR is to restrict the proliferation of missiles, complete rocket systems, unmanned air vehicles, and related

Page 67: Answer Key & Exp for GS Prelim Test-3 held on 2nd Dec’ 2018 · Commission report 1930, the 3 Round Table Conferences and the White Paper on Constitutional Reforms. What is called

Page 67 of 74

technology for those systems capable of carrying a 500 kilogram payload at least 300 kilometres, as well as systems intended for the delivery of weapons of mass destruction (WMD). Value Addition In July 2016, India became the 35th full member of the Missile Technology Control Regime (MTCR). It would be mutually beneficial in the furtherance of international non-proliferation objectives. It marked India’s first entry into any multilateral export control regime. Source: Vajiram & Ravi Institute Current Affairs Magazine: May 2018; Section: Science & Technology; Page no. 95 88. Regarding the Framework Convention on Tobacco Control (FCTC),

consider the following statements: (1) The FCTC, formed in 2015, is the first international treaty to be

agreed under Centres for Disease Control and Prevention (CDC). (2) Its sessions are held every 2 years and India is a party to it. Which of the statements given above is/are correct? (a) 1 only (b) 2 only (c) Both 1 and 2 (d) Neither 1 nor 2

Answer: (b) Explanation: Statement 1 is incorrect The Framework Convention on Tobacco Control (FCTC) formed in 2005, is first international treaty to be agreed under WHO auspices and has successfully helped to co-ordinate & energizes the global struggle against tobacco. Statement 2 is correct: Its sessions are held at every 2 years. The treaty provides for establishment of an international tracking system, passing of dissuasive law enforcement measures and enhancing international cooperation to address the challenges posed by tobacco menace. India is a party to WHO FCTC. Under Article 15 of the WHO FCTC, the protocol is applicable to both smoking and chewing or smokeless tobacco (SLT) forms. Source: Vajiram & Ravi Institute Current Affairs Magazine: May 2018; Section: Social Justice; Page No. 24 89. Recently, the ‘Health-care Access and Quality (HAQ)’ Index has been

released. With reference to it, consider the following statements: (1) The Index is created by the World Health Organization in association

with the World Bank. (2) India ranks lower than Bangladesh, Sudan and Equatorial Guinea. (3) India has improved its rank vis-a-vis its BRICS peers Brazil, Russia,

China and South Africa on the HAQ Index. Which of the statements given above is/are correct? (a) 1 and 2 only

Page 68: Answer Key & Exp for GS Prelim Test-3 held on 2nd Dec’ 2018 · Commission report 1930, the 3 Round Table Conferences and the White Paper on Constitutional Reforms. What is called

Page 68 of 74

(b) 2 only (c) 1, 2 and 3 (d) None of the above

Answer: (b) Explanation: Statement 1 is incorrect. Health-care Access and Quality (HAQ) index is created by the Global Burden of Disease study and published in the medical journal The Lancet. The index is based on 32 causes of death considered preventable with effective medical care. Statement 2 is correct. India has improved its ranking on a global health-care access and quality (HAQ) index from 153 in 1990 to 145 in 2016 (with a score of 41.2 points), yet ranks lower than neighbouring Bangladesh and even sub-Saharan Sudan and Equatorial Guinea. Statement 3 is incorrect. Despite improvements in health-care access and quality, India lags way behind its BRICS peers Brazil, Russia, China and South Africa on the HAQ index, but matches China in the disparity in health-care access and quality between states. Source: Vajiram & Ravi Institute Current Affairs Magazine: May 2018; Section: Social Justice; Page No. 27 90. The Ministry of Human Resource Development has launched ‘Samagra

Shiksha’ - an integrated scheme for school education. Which of the following statements is/are correct in this regard? (1) One of the most important provisions of the scheme is to fill the

vacant teaching posts and to give training to untrained teachers. (2) It subsumes the three Schemes of Sarva Shiksha Abhiyan, Rashtriya

Madhyamik Shiksha Abhiyan, and Teacher Education. Select the correct answer using the code given below: (a) 1 only (b) 2 only (c) Both 1 and 2 (d) Neither 1 nor 2

Answer: (b) Explanation: The main outcomes of the Scheme are envisaged as Universal Access, Equity and Quality, promoting Vocationalisation of Education, and strengthening of Teacher Education Institutions (TEIs). Statement 2 is correct. It envisages the ‘school’ as a continuum from pre-school, primary, upper primary, secondary to senior secondary levels and subsumes the three erstwhile centrally sponsored schemes i.e. Sarva Shiksha Abhiyan (SSA), Rashtriya Madhyamik Shiksha Abhiyan (RMSA) and Teacher Education(TE).

Page 69: Answer Key & Exp for GS Prelim Test-3 held on 2nd Dec’ 2018 · Commission report 1930, the 3 Round Table Conferences and the White Paper on Constitutional Reforms. What is called

Page 69 of 74

Statement 1 is incorrect. The scheme does not have a provision for filling vacant teaching posts or training untrained teachers, who numbered 15 lakh according to 2017 NSSO data. Source: Vajiram & Ravi Institute Current Affairs Magazine: May 2018; Section: Social Justice; Page No. 31 91. According to ‘The Status of Trace and Toxic metals in Indian Rivers 2018’

Report, major heavy metals polluting the Ganga river are: (1) Chromium (2) Copper (3) Nickel (4) Lead (5) Iron Select the correct answer using the code given below: (a) 1 and 2 only (b) 1, 2 and 3 only (c) 4 and 5 only (d) 1, 2, 3, 4 and 5

Answer: (d) Explanation: The ‘Status of Trace and Toxic metals in Indian Rivers 2018’ is released by Central Water Commission. According to the Status of trace and toxic metals in Indian rivers 2018 report, major heavy metals polluter found in Ganga River are chromium, copper, nickel, lead and iron. Source: http://www.indiaenvironmentportal.org.in/content/454984/status-of-trace-and-toxic-metals-in-indian-rivers-2018/ 92. The term ‘Project Trident’ is sometimes seen in the news in the context of

(a) India & Maldives (b) India & WTO (c) India & Mauritius (d) India & World Bank

Answer: (c) Explanation: India would support the National Coast Guard of Mauritius in augmenting its capacity through 'Project Trident'. Source: diplomacybeyond.com/articles/india-mauritius-relationship-indian-ocean-region/ 93. ‘Cochlear Implant’ is a modern technology to help young children who have

'severe to profound deafness in both ears.' In this context, consider the following statements: (1) The Cochlear Implant directly provides electrical stimulation to the

nerve endings in the cochlea thus enabling a child to hear.

Page 70: Answer Key & Exp for GS Prelim Test-3 held on 2nd Dec’ 2018 · Commission report 1930, the 3 Round Table Conferences and the White Paper on Constitutional Reforms. What is called

Page 70 of 74

(2) In India, the Cochlear Implant programme is being implemented by the Ministry of Health and Family Welfare.

Which of the statements given above is/are correct? (a) 1 only (b) 2 only (c) Both 1 and 2 (d) Neither 1 nor 2

Answer: (a) Explanation: Hearing is crucial to language development in children which generally takes place between ages one and three. The cochlear implant is a prosthetic device, a part of which is surgically implanted inside the cochlea. Cochlear implants have been found to be beneficial for children and adults with severe to profound hearing loss and steeply sloping hearing loss who do not benefit adequately with hearing aids but have an intact auditory nerve. While a hearing aid provides amplified sound energy to the ear, the cochlear implant directly provides electrical stimulation to the nerve endings in the cochlea. It is being implemented by Ministry of Social Justice and Empowerment. Source: https://www.nidcd.nih.gov/health/cochlear-implants 94. With reference to the 'Exercise Ekatha', which of the following statements

is/are correct? (1) It is an annual naval exercise between India and Vietnam. (2) It aims to increase India's strategic capacity in South China Sea. Select the correct answer using the code given below: (a) 1 only (b) 2 only (c) Both 1 and 2 (d) Neither 1 nor 2

Answer: (d) Explanation: The 2nd Asymmetric Warfare Training Exercise 'Ekatha 2018' was held in Maldives in May 2018. 'Exercise Ekatha' is an exercise between India and Maldives. It aims to increase India's strategic capacity in the Indian Ocean. Source: https://indianexpress.com/article/india/indian-ship-deployed-to-undertake-joint-surveillance-of-maldives-exclusive-economic-zone-5173232/ 95. Doppler radars provide precise information about which of the following?

(1) Thunderstorms (2) Dust storms (3) Hailstorms

Page 71: Answer Key & Exp for GS Prelim Test-3 held on 2nd Dec’ 2018 · Commission report 1930, the 3 Round Table Conferences and the White Paper on Constitutional Reforms. What is called

Page 71 of 74

(4) Rainfall (5) Wind patterns Select the correct answer using the code given below: (a) 1, 3 and 4 only (b) 2, 3 and 4 only (c) 1 and 4 only (d) 1, 2, 3, 4 and 5

Answer: (d) Explanation: Doppler radars provide precise information about thunderstorms, dust storms, hailstorms, rainfall and wind patterns. With a radius of 250km, they help in issuing now-casts 2-3 hours prior to severe weather events. The first Doppler radar was installed in Chennai in 2002. Its need became more compelling after the 2005 Mumbai floods. There are currently 27 Doppler radars in the country. Source: https://en.wikipedia.org/wiki/Doppler_radar 96. This National Park is situated in the foothills of the Sub-Himalayan belt. It

has the glory of being India’s oldest and most prestigious National Park. It is also being honoured as the place where Project Tiger was first launched in 1973'. This unique National Park is known as: (a) Jim Corbett National Park (b) Namdapha National Park (c) Jaldapara National Park (d) Manas National Park

Answer: (a) Explanation: Jim Corbett National Park is situated in the foothills of the Sub-Himalayan belt in Nainital districts of Uttarakhand state in India. Established in the year 1936 as Hailey National Park. It covers an area of 521 sq. km and together with the neighbouring Sonanadi Wildlife Sanctuary and Reserve Forest areas forms the Corbett Tiger Reserve. Corbett is one of the richest bird regions of the Country and has been declared as an ‘Important Bird Area’ (IBA) by Birdlife International. Source: Vajiram & Ravi Institute Current Affairs Magazine: May 2018; Section: Environment and Biodiversity; Page No.106 97. The ‘Joint Comprehensive Plan of Action (JCPOA)’ which was in the news

recently is associated with: (a) Nuclear Agreement (b) Boundary Agreement (c) Trade Agreement (d) Environmental Agreement

Answer: (a) Explanation:

Page 72: Answer Key & Exp for GS Prelim Test-3 held on 2nd Dec’ 2018 · Commission report 1930, the 3 Round Table Conferences and the White Paper on Constitutional Reforms. What is called

Page 72 of 74

The Joint Comprehensive Plan of Action (JCPOA) known commonly as the Iran nuclear deal or Iran deal, is an agreement on the nuclear program of Iran reached in Vienna on 14 July 2015 between Iran, the P5+1 (the five permanent members of the United Nations Security Council—China, France, Russia, United Kingdom, United States—plus Germany), and the European Union. On 8 May 2018, US President Trump announced that the United States would withdraw from the agreement. Following the U.S.'s withdrawal, the EU enacted an updated blocking statute on 7 August 2018 to nullify US sanctions on countries trading with Iran. Source: Vajiram & Ravi Institute Current Affairs Magazine: May 2018; Section: International Relations; Page No. 39 98. The terms 'Boto', 'Tucuxi', 'Baiji' are seen in news sometimes. These are

related to which of the following? (a) These are different species of flower found in Western Ghat (b) These are the names of different species of Dolphins (c) These are fruit bats which hosts Nipah virus (d) These are different species of butterflies found in Eastern Himalaya

Answer: (b) Explanation: Looking towards the conservation of Indus Dolphins -- one of the world's rarest mammals -- Punjab government along with WWF-India is conducting a first organised census. Found only in India and Pakistan, the Indus Dolphins are confined to only 185 km stretch between Talwara and Harike Barrage in India's Beas river in Punjab. Boto also known as the pink river dolphin ,can only live in freshwater. It is found throughout much of the Amazon and Orinoco River Basins in Bolivia, Brazil, Colombia, Ecuador, Guyana, Peru, and Venezuela. The Tucuxi lives in both salt- and freshwater and is found on the east coast of Central and South America. The Yangtze river dolphin, or Baiji, was declared "functionally extinct" in 2006 after scientists failed to locate any remaining animals. The Endangered Ganges river dolphin, or Susu, can only live in freshwater and are naturally blind. Source: Vajiram & Ravi Institute Current Affairs Magazine: May 2018; Section: Environment and Biodiversity; Page No.100 99. North Korea which was in the news recently is located between which of

the following Seas? (a) Korea Bay and Sea of Japan (b) Yellow Sea and East China Sea (c) Korea Bay and Yellow Sea (d) East China Sea and Sea of Japan

Answer: (a) Explanation:

Page 73: Answer Key & Exp for GS Prelim Test-3 held on 2nd Dec’ 2018 · Commission report 1930, the 3 Round Table Conferences and the White Paper on Constitutional Reforms. What is called

Page 73 of 74

Recently, North Korea and the U.S. agreed at a summit in Singapore to “work toward” denuclearization. North Korea is located between Korea Bay and the Sea of Japan.

Source: Vajiram & Ravi Institute Current Affairs Magazine: May 2018; Section: International Relations; Page No. 39 100. With reference to Mission Innovation (MI), which of the following

statements is/are correct? (1) Its focus is on accelerating clean energy innovation and boosting

public research and development spending in the field of clean energy.

(2) It is a global initiative of 24 countries which includes India and the European Union.

(3) It was announced during the Paris Summit to undertake ambitious efforts to combat climate change.

Select the correct answer using the code given below: (a) 1 only (b) 1 and 3 only (c) 2 and 3 only (d) 1, 2 and 3

Answer: (d) Explanation: Mission Innovation (MI) is a global initiative of 24 countries and the European Union to dramatically accelerate global clean energy innovation. As part of the initiative, participating countries have committed to seek to double their governments’ clean energy research and development (R&D) investments over five years, while encouraging greater levels of private sector investment in transformative clean energy technologies. India is a founding member of Mission Innovation and part of the Steering Committee besides coleader of innovation challenges on smart grids, off grids and sustainable bio-fuels Source: Vajiram & Ravi Institute Current Affairs Magazine: May 2018; Section: Environment and Biodiversity; Page No.100

Page 74: Answer Key & Exp for GS Prelim Test-3 held on 2nd Dec’ 2018 · Commission report 1930, the 3 Round Table Conferences and the White Paper on Constitutional Reforms. What is called

Page 74 of 74